Крок 3 - Медицина 2016 осінь (буклет)

1 / 200
Військовослужбовець отримав вогнепальне поранення стегна з пошкодженням стегнової кістки. На догоспітальному етапі проведено знеболювання, зупинена кровотеча, кінцівка іммобілізована. Під час транспортування з’явився біль у грудній клітці, задишка, тахікардія. Що стало причиною погіршення стану пораненого? The serviceman received a gunshot wound to the thigh with damage to the femur. At the pre-hospital stage, anesthesia was administered, the bleeding was stopped, the limb was immobilized. During transportation, chest pain appeared, shortness of breath, tachycardia. What caused the deterioration of the injured person's condition?

Жирова емболія Fat embolism

Інфаркт міокарда Myocardial infarction

Тромбоемболія гілок легеневої артерії Thromboembolism of the branches of the pulmonary artery

Набряк легенів Pulmonary edema

Напад бронхіальної астми Bronchial asthma attack

2 / 200
До приймального відділення військового госпіталю доставлено військовослужбовця з вогнепальним пораненням в ділянку черевної порожнини. Шкіра бліда, AT- 90/60 мм рт.ст., Ps- 100/хв. Симптоми подразнення очеревини сумнівні. Яке дослідження треба виконати для уточнення діагнозу? A serviceman with a gunshot wound in the abdominal cavity was brought to the reception department of the military hospital. Pale skin, AT- 90/60 mm Hg, Ps- 100/ min. Symptoms of peritoneal irritation are doubtful. What research should be performed to clarify the diagnosis?

Оглядова рентгеноскопія органів черевної порожнини Rentoscopy of abdominal organs

Фіброгастродуоденоскопія Fibrogastroduodenoscopy

Лапаротомія Laparotomy

УЗД органів черевної порожнини Ultrasound of abdominal organs

Комп’ютерна томографія органів черевної порожнини Computer tomography of abdominal organs

3 / 200
До медичної роти доставлено постражда-лого з тяжким вогнепальним пораненням нижньої третини правого стегна. Кістка роздроблена. Магістральні судини пошкоджені. Кінцівка висить на клапті м’яких тканин. До якого виду медичної допомоги відноситься проведення відсікання кінцівки, яка висить на клапті м’яких тканин? A victim with a severe gunshot wound to the lower third of the right thigh was brought to the medical unit. The bone is crushed. The trunk vessels are damaged. The limb hangs on a flap of soft tissue. To What type of medical care does amputation of a limb that hangs on a flap of soft tissue refer to?

Першої медичної допомоги First aid

Першої лікарської допомоги First aid

Долікарської (фельдшерської) медичної допомоги Paramedic (paramedic) medical care

Кваліфікованої медичної допомоги Qualified medical care

Спеціалізованої медичної допомоги Specialized medical care

4 / 200
Хвора 50-ти років поступила в клініку зі скаргами на напади сильного головного болю, запаморочення, відчуття пульсації в голові, пітливість, м’язову слабкість. Під час нападу підвищується артеріальний тиск до 280/170 мм рт.ст. Напади тривають до 30 хвилин. Останнім часом почастішали. За даними УЗД - визначається об’ємне утворення правого наднирника. Який препарат найбільш показаний хворій? A 50-year-old patient was admitted to the clinic with complaints of attacks of severe headache, dizziness, a feeling of pulsation in the head, sweating, muscle weakness. During the attack blood pressure rises to 280/170 mm Hg. Attacks last up to 30 minutes. Recently, they have become more frequent. According to ultrasound, a volume formation of the right adrenal gland is determined. What drug is most indicated for the patient?

Фентоламін в/в Phentolamine IV

Папаверин в/в Papaverine IV

Пентамін в/в Pentamine IV

Дибазол в/в Dibazol IV

Фуросемід в/в Furosemide IV

5 / 200
У пацієнта з симптомами відсутності кровообігу на ЕКГ встановлена асистолія. Першочерговим терапевтичним заходом має бути: A patient with symptoms of lack of blood circulation on the ECG has asystole. The first therapeutic measure should be:

Введення атропіну Atropine administration

Введення хлориду кальцію Introduction of calcium chloride

Введення адреналіну Injection of adrenaline

Введення лідокаїну Lidocaine administration

Електрична дефібриляція Electrical defibrillation

6 / 200
Травмований мотоцикліст лежить на проїжджій частині дороги. Ліве стегно його деформоване в нижній третині, через зовнішню рану фонтанує кров. Де і який кровоспинний джгут необхідно застосувати? The injured motorcyclist is lying on the road. His left thigh is deformed in the lower third, blood is pouring out of the external wound. Where and what kind of tourniquet should be applied?

Венозний джгут безпосередньо над раною Venous tourniquet directly above the wound

Артеріальний джгут в середній третині стегна Arterial tourniquet in the middle third of the thigh

Артеріальний джгут безпосередньо над раною стегна Arterial tourniquet directly above thigh wound

Артеріальний джгут в верхній третині стегна Arterial tourniquet in the upper third of the thigh

Венозний джгут на верхню третину гомілки Venous tourniquet on the upper third of the lower leg

7 / 200
Реанімаційна бригада швидкої медичної допомоги прибула на виклик до опіатного наркомана 23-х років. Об’єктивно: свідомість відсутня, ціаноз, брадипное, брадикардія. Що терміново показано хворому? The resuscitation team of emergency medical aid arrived on call to a 23-year-old opiate addict. Objectively: unconsciousness, cyanosis, bradypnea, bradycardia. What is urgently shown to the patient ?

Штучна вентиляція легень Artificial lung ventilation

Оксигенотерапія через маску Oxygenotherapy through a mask

Інфузійна терапія Infusion therapy

Призначення налорфіну Nalorphine Prescription

Форсований діурез Forced diuresis

8 / 200
На стаціонарному лікуванні перебуває вагітна жінка 33-х років. Строк вагітності 34 тижні. Впродовж останніх 2-х діб скаржиться на головний біль, запаморочення, відчуття тяжкості в ділянці потилиці, зорові розлади. Мають місце набряки. AT-160/120 мм рт.ст., ЧСС- 88/хв. В сечі: протягом доби зростаюча протеїнурія до 3,2 г/л. Діурез знижений. Яка найбільш імовірна патологія зумовила цю симптоматику? A 33-year-old pregnant woman is undergoing inpatient treatment. She is 34 weeks pregnant. For the past 2 days, she has been complaining of a headache, dizziness, and a feeling of heaviness in the area occiput, visual disturbances. Swelling occurs. AT-160/120 mm Hg, heart rate- 88/min. In the urine: during the day, increasing proteinuria up to 3.2 g/l. Diuresis is reduced. What is the most probable pathology that caused this symptoms?

Прееклампсія середньої тяжкості Preeclampsia of moderate severity

Іостре порушення мозкового кровообігу Acute cerebrovascular accident

Гіпертонічний криз Hypertensive crisis

Еклампсія Eclampsia

Тяжка прееклампсія Severe preeclampsia

9 / 200
У хворого, який перебуває в гіпергліке-мічній комі, через 4 години після початку інсулінотерапії раптово розвинулися тахікардія, м’язова гіпотонія; AT- 80/40 мм рт.ст. На ЕКГ: подовження інтервалу QT, зниження S — Т, розширення і сплощення Т. Який найбільш імовірний діагноз? A patient in a hyperglycemic coma suddenly developed tachycardia and muscle hypotension 4 hours after the start of insulin therapy; AT- 80/40 mm Hg On the ECG: prolongation of the QT interval, reduction of S — T, expansion and flattening of T. What is the most likely diagnosis?

Гіпокаліємія Hypokalemia

Гіпоглікемія Hypoglycemia

Гіперглікемія Hyperglycemia

Гіповолемія Hypovolemia

Гіперкаліємія Hyperkalemia

10 / 200
До приймального відділення лікарні звернувся постраждалий у стані легкого алкогольного сп’яніння зі скаргами на порушення зору, що виникли після вживання спиртного. Найбільш імовірний діагноз: A victim in a state of mild alcohol intoxication came to the reception department of the hospital with complaints of visual disturbances that occurred after drinking alcohol. The most likely diagnosis:

Гострий напад глаукоми Acute attack of glaucoma

Отруєння метанолом Methanol poisoning

Отруєння етанолом Ethanol poisoning

Отруєння етиленгліколем Ethylene glycol poisoning

Гостре порушення мозкового кровообігу Acute cerebrovascular accident

11 / 200
Постраждалий вилучений з палаючої будівлі з опіками ІІ-ІІІА ступеня 30% поверхні тіла. Рівень свідомості - кома І. Найбільш імовірною причиною коматозного стану є: The victim was removed from the burning building with II-IIIA degree burns on 30% of the body surface. The level of consciousness is coma I. The most likely cause of the comatose state is:

Важкий опіковий шок Severe burn shock

Психоемоційний стрес Psycho-emotional stress

Отруєння парами синільної кислоти Poisoning by vapors of hydrocyanic acid

Отруєння чадним газом Monoxide poisoning

Отруєння ціанідами Cyanide poisoning

12 / 200
У пацієнта 40-ка років, котрий впродовж 17-ти років хворіє на неспецифічний виразковий коліт, при останньому ендоскопічному дослідженні виявлено множинні запальні псевдополіпи у всіх відділах товстої кишки. Результати біопсії показали наявність в поліпах дисплазії ІІ-ІІІ ст. Яке лікування необхідно застосувати в даного хворого? In a 40-year-old patient who has been suffering from non-specific ulcerative colitis for 17 years, the last endoscopic examination revealed multiple inflammatory pseudopolyps in all sections of the colon. The results of the biopsy showed the presence of grade II-III dysplasia in the polyps. What treatment should be applied to this patient?

Рентгенотерапія X-ray therapy

Медикаментозне Medical

Хірургічне Surgical

Трансендоскопічне видалення поліпів Transendoscopic removal of polyps

Хіміотерапія Chemotherapy

13 / 200
Хлопчик 6-ти років скаржиться на загальну слабкість, генералізоване свербіння та почервоніння шкіри, гостре підвищення температури тіла до 38°С. З анамнезу: перший день хвороби. Протягом кількох годин після обробки невеликого садна іхтіоловою маззю з’явилися поширені гіперемовані папульозно-уртикарні висипання, що виступають над поверхнею шкіри в ділянці передпліччя, живота, стегон, гомілок. Вкажіть найбільш імовірний діагноз: A 6-year-old boy complains of general weakness, generalized itching and reddening of the skin, a sharp rise in body temperature up to 38°C. From the anamnesis: the first day of illness. During a few hours after treating a small wound with ichthyol ointment, widespread hyperemic papular-urticarial rashes appeared, protruding above the surface of the skin in the area of ​​the forearm, abdomen, thighs, and lower legs. Specify the most likely diagnosis:

Генералізована кропивниця Generalized urticaria

Токсикодермія Toxicoderma

Алергічний контактний дерматит Allergic contact dermatitis

Набряк Квінке Quincke edema

Атопічний дерматит - нейродерміт, період загострення Atopic dermatitis - neurodermatitis, period of exacerbation

14 / 200
Постраждалий 42-х років, виявлений на місці ДТП, скаржиться на біль у ділянці таза та правого стегна. Об’єктивно: загальмований, стогне, шкіра бліда, язик сухий. ЧД- 28/хв. Праве стегно деформоване, набрякле, у верхній його третині патологічна рухливість, крепітація кісткових уламків. Великий крововилив, набряк та біль під час пальпації в правих пахвинній, здухвинній та сідничній ділянках. Ps- 112/хв., AT- 80/45 мм рт.ст. Якому ступеню травматичного шоку відповідає стан потерпілого? A 42-year-old victim found at the scene of an accident complains of pain in the pelvis and right thigh. Objectively: slowed down, moaning, pale skin, tongue dry. ChD- 28/min. The right thigh is deformed, swollen, in its upper third there is pathological mobility, crepitation of bone fragments. Large hemorrhage, swelling and pain during palpation in the right inguinal, pubic and buttock areas. Ps- 112/min. AT- 80/45 mm Hg. What degree of traumatic shock does the victim's condition correspond to?

II II

І And

- -

III III

IV IV

15 / 200
Дитина 6-ти років госпіталізована до дитячого відділення з бронхопневмонією. Страждає на атопічний дерматит. Після внутрішньом’язового введення ампіциліну з’явилися відчуття стиснення у грудях, запаморочення, різка блідість, ціаноз, холодний піт, прискорене шумне дихання. Який з перерахованих препаратів слід ввести в маніпуляційній негайно? A 6-year-old child was hospitalized in the children's ward with bronchopneumonia. He suffers from atopic dermatitis. After intramuscular administration of ampicillin, chest tightness, dizziness, sharp pallor, cyanosis, cold sweat, accelerated noisy breathing. Which of the listed drugs should be administered in the manipulation room immediately?

Розчин еуфіліну внутрішньовенно Euphilin intravenous solution

Розчин преднізолону внутрішньовенно Prednisone intravenous solution

Розчин строфантину внутрішньовенно Strophanthine intravenous solution

Розчин адреналіну підшкірно Solution of adrenaline subcutaneously

Розчин тавегілу внутрішньовенно Tavegil intravenous solution

16 / 200
Хвора 20-ти років захворіла гостро, підвищилась температура до 39,5°С, сильний головний біль, спостерігалось кілька разів блювання. Об’єктивно: на шкірі геморагічний висип, ригідність потиличних м’язів, симптоми Керніга та Брудзинського позитивні. Якою повинна бути тактика лікаря? A 20-year-old patient became acutely ill, the temperature rose to 39.5°C, severe headache, vomiting was observed several times. Objectively: hemorrhagic skin rash, stiffness of the occipital muscles, Kernig's and Brudzinsky's symptoms are positive. What should be the doctor's tactics?

Залишити для спостереження вдома Leave for observation at home

Призначити антибактеріальну терапію Prescribe antibacterial therapy

Негайна госпіталізація в інфекційний стаціонар Immediate hospitalization in an infectious disease hospital

Викликати на консультацію невропатолога Call a neurologist for consultation

Зробити загальний аналіз крові Do a general blood test

17 / 200
Хлопчику 12 років. Знаходиться в клініці з приводу цукрового діабету в фазі декомпенсації. Отримав перед сніданком 10 ОД і перед обідом 6 ОД простого інсуліну. Через ЗО хвилин після обіду хлопчик втратив свідомість, з’явились судоми. Об’єктивно: шкіра волога, бліда, тризм щелеп. Тони серця приглушені, Ps- 100/хв., слабкого наповнення, AT- 90/50 мм рт.ст., дихання поверхневе, 28/хв. Першочергова допомога в даній ситуації включає: The boy is 12 years old. He is in the clinic for diabetes in the decompensation phase. He received 10 units before breakfast and 6 units of simple insulin before lunch. Thirty minutes after lunch the boy lost consciousness, convulsions appeared. Objectively: the skin is moist, pale, trismus of the jaws. Heart sounds are muffled, Ps- 100/min., weak filling, AT- 90/50 mm Hg, shallow breathing, 28 /min. First aid in this situation includes:

Введення інсуліну Insulin administration

Введення мезатону Introduction of mesotone

Введення преднізолону Prednisolone administration

Введення глюкози Introduction of glucose

Оксигенотерапія Oxygenotherapy

18 / 200
Дівчинка 4-х років скаржиться на загальну слабкість, кашель. Вночі раптово підвищилася температура до 39,8°С, хвора стала неспокійною. Гавкаючий кашель, шумне свистяче дихання з втягуванням над- та підключичних западин, міжреберних проміжків, шкіра бліда. Контактувала з хворими на ГРВІ сестрами. Що необхідно призначити дитині в першу чергу? A 4-year-old girl complains of general weakness, cough. At night, the temperature suddenly rose to 39.8°C, the patient became restless. Barking cough, noisy whistling with retraction of supraclavicular and subclavian hollows, intercostal spaces, pale skin. Contacted nurses with SARS. What should be prescribed to the child first of all?

Астмопент, діазолін Astmopent, diazolin

Антибіотики, димедрол Antibiotics, diphenhydramine

Папаверин, вітамін С Papaverine, vitamin C

Преднізолон, гарячі ніжні ванни Prednisone, hot gentle baths

Дибазол, еуфілін Dibazol, Euphilin

19 / 200
До лікарні бригадою ШМД доставлено жінку 32-х років. З анамнезу: протягом останніх 2х років перебувала на диспансерному обліку з приводу аутоімунної гемолітичної анемії. Об’єктивно: стан середньої важкості, температура - 37,3°С, шкіра лимонно-жовтого кольору, Ps- 98/хв., АТ-80/55 мм рт.ст., збільшена селезінка. У крові: ер,- 2,6•1012/л, Нb- 70 г/л, КП- 0,94, лейк,-6,1•109/л, ШОЕ-19 мм/год, непрямий білірубін - 58 мкмоль/л. Що необхідно призначити в першу чергу? A 32-year-old woman was brought to the hospital by the medical team. From the anamnesis: for the past 2 years, she has been under medical records for autoimmune hemolytic anemia. Objectively: the condition of moderate severity, temperature - 37.3°С, lemon-yellow skin, Ps- 98/min., BP-80/55 mm Hg, enlarged spleen. Blood: er, - 2.6•1012/ l, Hb - 70 g/l, KP - 0.94, leuk, -6.1•109/l, ESR - 19 mm/h, indirect bilirubin - 58 μmol/l. What should be prescribed first of all?

Трансфузія консервованої крові Transfusion of canned blood

Інтерферон Interferon

Еритроцитарна маса Erythrocyte mass

Преднізолон Prednisone

Ферум-лек Ferum-lek

20 / 200
Хворий 50-ти років після аварії на виробництві був госпіталізований із скаргами на металевий присмак та печіння у роті, нудоту, слинотечу, багаторазове блювання, пронос з домішками слизу та крові, біль у епігастральній та поперековій ділянках. Об’єктивно: стан важкий, виразковий стоматит, гінгівіт, некроз слизової оболонки носа, набряк гортані. AT- 150/100 мм рт.ст., Ps- 48/хв., тони серця глухі. Поліурія, гіпостенурія, альбумінурія, гематурія. Який антидот слід ввести хворому? A 50-year-old patient was hospitalized after an industrial accident with complaints of a metallic taste and burning in the mouth, nausea, drooling, repeated vomiting, diarrhea with mucus impurities and of blood, pain in the epigastric and lumbar areas. Objectively: the condition is severe, ulcerative stomatitis, gingivitis, necrosis of the nasal mucosa, swelling of the larynx. AT- 150/100 mm Hg, Ps- 48/min, heart sounds are dull Polyuria, hyposthenuria, albuminuria, hematuria. What antidote should be administered to the patient?

Атропіну сульфат Atropine sulfate

Дієтиоксим Dietioxime

Антарсин Antarsyn

Хромосмон Chromosmon

Унітіол Unithiol

21 / 200
До реанімаційного відділення доставлена хвора 48-ми років без свідомості. Шкіра холодна, бліда, суха, акроціаноз. Щільний набряк обличчя, тіла, гомілок, температура тіла - 33°С. Сухожильні рефлекси мляві. Ps- 50/хв., малий, ритмічний. AT- 90/60 мм рт.ст. Тони серця різко послаблені. Дихання рідке, глибоке. У крові: гіпохромна анемія, гіперхолестерінемія, високий рівень Т4 крові. Який препарат необхідно призначити в першу чергу? A 48-year-old unconscious patient was brought to the intensive care unit. The skin is cold, pale, dry, acrocyanosis. Dense swelling of the face, body, legs, body temperature - 33 °С. Tendon reflexes are sluggish. Ps- 50/min, small, rhythmic. AT- 90/60 mm Hg. Heart tones are sharply weakened. Breathing is rare, deep. In the blood: hypochromic anemia, hypercholesterolemia, high level of T4 in the blood What drug should be prescribed first?

Глюкоза Glucose

Трийодтиронін Triiodothyronine

Анаприлін Anaprilin

Мерказоліл Mercazolil

Преднізолон Prednisone

22 / 200
У хворої 42-х років гострий лівобічний пієлонефрит ускладнився бактеріальним шоком та гострою нирковою недостатністю. Дві доби тому настала поліурична стадія. Які зміни в організмі слід попередити в першу чергу? In a 42-year-old patient, acute left-sided pyelonephritis was complicated by bacterial shock and acute renal failure. Two days ago, the polyuric stage began. What changes in the body should be prevented first of all ?

Анемія Anemia

Гіпокаліємія Hypokalemia

Гіпоальбумінемія Hypoalbuminemia

Набряковий синдром Oedema syndrome

Гіперазотемія Hyperazotemia

23 / 200
Пологи п’яті. Пологова діяльність - перейми по 35-40 сек., через 3-4 хвилини. Положення плоду поздовжнє, передлежить голівка малим сегментом у вході в малий таз. Серцебиття плоду ясне, ритмічне, 132-140/хв. Вагінально: шийка матки згладжена, розкриття 8 см. Плідний міхур відсутній. Передлежить голівка на II площині малого тазу, справа допереду визначається перенісся і надбрівні дуги. Який імовірний діагноз? Fifth birth. Labor activity - pauses for 35-40 seconds, after 3-4 minutes. The position of the fetus is longitudinal, the head is presented by a small segment at the entrance to small pelvis. The heartbeat of the fetus is clear, rhythmic, 132-140/min. Vaginal: the cervix is ​​smoothed, the opening is 8 cm. The fetal bladder is absent. The head lies on the II plane of the small pelvis, the bridge of the nose and the brow ridges are defined from the right to the front. What is the probable diagnosis?

Другий період пологів, лобне передле-жання Second period of childbirth, frontal presentation

Перший період пологів, лицеве передле-жання First period of childbirth, facial presentation

Перший період пологів, сідничне пере-длежання First period of labor, breech presentation

Перший період пологів, передньоголовне передлежання First period of labor, anterior cephalic presentation

Перший період пологів, лобне передле-жання First period of childbirth, frontal presentation

24 / 200
Чоловік 78-ми років викликав швидку допомогу з приводу блювання, здуття живота, наявності болючого грижового випинання у паху, яке не вправляється у черевну порожнину зі вчорашнього дня. Під час огляду лікарем швидкої допомоги грижове випинання вправилося, біль пройшов. Які дії лікаря? A 78-year-old man called an ambulance due to vomiting, abdominal distension, the presence of a painful hernial protrusion in the groin, which has not been inserted into the abdominal cavity since yesterday. Under during the examination by the emergency doctor, the hernial protrusion was relieved, the pain went away. What actions did the doctor take?

Призначити амбулаторне обстеження Schedule outpatient examination

Рекомендувати хірургічне лікування у плановому порядку Recommend surgical treatment as planned

Протягом години спостерігати за динамікою клінічної картини Over the course of an hour, observe the dynamics of the clinical picture

Призначити дієту і холод на живіт Prescribe a diet and a cold on the stomach

Доставити хворого до стаціонару Deliver the patient to the hospital

25 / 200
Чоловіка 40-ка років доставлено до приймального відділення. Відзначається запах алкоголю з рота, відкривання очей та мовні відповіді відсутні, нецілеспрямова-ний рух на біль. Об’єктивно: дихання часте (>40/хв.), поверхневе, AT- 90/40 мм рт.ст., Ps- 112/хв. Що треба зробити в першу чергу? A 40-year-old man was brought to the reception department. The smell of alcohol from the mouth is noted, there are no eye opening and speech responses, non-targeted movement due to pain. Objectively : breathing is frequent (>40/min.), superficial, AT- 90/40 mm Hg, Ps- 112/min. What should be done first of all?

Внутрішньовенно бемегрід Intravenous bemegrid

Зондове промивання шлунка Gastric tube lavage

Позаниркові методи детоксикації (гемодіаліз, гемосорбція) Extrarenal detoxification methods (hemodialysis, hemosorption)

Форсований діурез Forced diuresis

Інтубація трахеї Tracheal intubation

26 / 200
Жінка 37-ми років, внаслідок тяжкої черепно-мозкової травми що була одержана у 20річному віці страждає на великі судомні напади. Дві години тому розпочалися генералізовані судомні пароксизми, протягом двох хвилин. У міжсудомний період свідомість порушена. Хвора не відповідає на запитання, реакція зіниць на світло в’яла, на больові подразники реагує слабо, виражена тахікардія. Оберіть найбільш адекватну терапію: A 37-year-old woman suffers from severe convulsive attacks as a result of a severe brain injury she received at the age of 20. Two hours ago, generalized convulsive paroxysms began, during two minutes. In the interconvulsive period, consciousness is disturbed. The patient does not answer questions, the reaction of the pupils to light is sluggish, she reacts weakly to painful stimuli, tachycardia is pronounced. Choose the most adequate therapy:

Протисудомна терапія та призначення анальгетиків Anticonvulsant therapy and prescription of analgesics

Боротьба з набряком мозку та призначення ноотропів Combating cerebral edema and prescribing nootropics

Боротьба з набряком мозку та протису-домна терапія Combating cerebral edema and protisus - home therapy

Протисудомні та антигістамінні засоби Anticonvulsants and antihistamines

Серцеві глікозиди та вітамінотерапія Cardiac glycosides and vitamin therapy

27 / 200
Пацієнт поступив в приймальне відділення психлікарні. Зловживає алкоголем 15 років, неодноразово лікувався. Останній запій тривав 2 місяці. З дні не вживав алкоголю, став збудженим, вночі не спав. Об’єктивно: шкірні покриви вологі, гіперемія обличчя, виражений тремор. Оглядається, під ліжком бачить щурів та павуків, намагається їх спіймати. Дезорієнтований у місці, часі. Препаратом вибору є: The patient was admitted to the reception department of a psychiatric hospital. He has been abusing alcohol for 15 years, he has been treated several times. The last binge lasted 2 months. He did not drink alcohol during the day, became agitated, did not sleep at night . Objectively: moist skin, hyperemia of the face, pronounced tremors. He looks around, sees rats and spiders under the bed, tries to catch them. Disoriented in place, time. The drug of choice is:

Седуксен Seduxen

Анальгін Analgin

Аспірин Aspirin

Вітамін С Vitamin C

Фінлепсин Finlepsin

28 / 200
На другу добу після субтотальної резекції щитоподібної залози з приводу дифузного токсичного зобу у хворої з’явилися тонічні судоми пальців ('рука акушера'), позитивні симптоми Хвостека, Вейса, Труссо. Яке ускладнення операції виникло? On the second day after subtotal resection of the thyroid gland due to diffuse toxic goiter, the patient developed tonic spasms of the fingers ('hand of the obstetrician'), positive symptoms of Khvostek, Weiss , Trousseau. What complication of the operation occurred?

Гіпопаратиреоз Hypoparathyroidism

Анафілактичний шок Anaphylactic shock

Асфіксія Asphyxia

Інфаркт міокарда Myocardial infarction

Тиреотоксичний криз Thyrotoxic crisis

29 / 200
У хворого на епілепсію в стаціонарі виник статус генералізованих тоніко-клонічних судом. Який з немедикаментозних заходів може бути використаний для покращення стану хворого? A patient with epilepsy in the hospital developed generalized tonic-clonic seizures. Which of the non-pharmacological measures can be used to improve the patient's condition?

Бальнеотерапія Balneotherapy

Магнітотерапія Magnetic therapy

Електрофорез комірцевої зони Electrophoresis of the collar zone

Су-джок терапія Su-jock therapy

Краніо-церебральна гіпотермія Cranio-cerebral hypothermia

30 / 200
Хворий 53-х років скаржаться на біль у ділянці прямої кишки при дефекації, яскравочервону кров на калових масах, яка з’являється на початку акту дефекації, закрепи. Загальний стан задовільний. Тони серця ритмічні, ЧСС- 74/хв. Живіт м’який, не болючий. Яке інструментальне дослідження найбільш інформативне для встановлення остаточного діагнозу? A 53-year-old patient complains of pain in the region of the rectum during defecation, bright red blood in the stool, which appears at the beginning of the act of defecation, constipation. General the condition is satisfactory. The heart sounds are rhythmic, the heart rate is 74/min. The abdomen is soft, not painful. What instrumental examination is the most informative for establishing the final diagnosis?

Фіброколоноскопія Fibrocolonoscopy

Пальцеве дослідження прямої кишки Digital rectal examination

Копрограма Coprogram

Ректоскопія Rectoscopy

Іригоскопія Irigoscopy

31 / 200
В консультацію звернулась жінка зі скаргами на біль у нижній частині живота, який виник раптово на 17-й день менструального циклу. Біль супроводжувався відчуттям тиску на пряму кишку. Об’єктивно: температура тіла - 36,9°С, Ps- 89/хв., AT-110/65 мм рт.ст. Вагінально: матка щільна, безболісна, рухома, нормальних розмірів. Додатки з лівої сторони помірно збільшені, болісні. При пункції черевної порожнини через заднє склепіння отримано 15 мл серозно-геморагічної рідини. Який найбільш імовірний діагноз? A woman came to the consultation with complaints of pain in the lower abdomen, which appeared suddenly on the 17th day of the menstrual cycle. The pain was accompanied by a feeling of pressure on the rectum. About Objectively: body temperature - 36.9°С, Ps - 89/min., AT-110/65 mm Hg. Vaginal: uterus is dense, painless, mobile, of normal size. Appendices on the left side are moderately enlarged, painful. During a puncture of the abdominal cavity through the posterior vault, 15 ml of serous-hemorrhagic fluid was obtained. What is the most likely diagnosis?

Гострий апендицит Acute appendicitis

Перекрут ніжки кісти яєчника Ovarian cyst pedicle torsion

Розрив кісти яєчника Rupture of an ovarian cyst

Позаматкова вагітність Ectopic pregnancy

Апоплексія яєчника Ovarian apoplexy

32 / 200
Хвора 28-ми років, вагітність ЗО тижнів, знаходилася вдома. Мала набряки гомілок 1 ступеня. Раптово поскаржилася на головний біль та втратила свідомість. На момент огляду констатовано опістотонус, голова закинута, зіниці розширені, дихання відсутнє, язика прикушено, пульс визначити важко. Яка стадія еклампсії у хворої? A 28-year-old patient, 3 weeks pregnant, was at home. She had swelling of the lower legs of the 1st degree. She suddenly complained of a headache and lost consciousness. At the time of the examination, opisthotonus was diagnosed , the head is thrown back, the pupils are dilated, there is no breathing, the tongue is bitten, it is difficult to determine the pulse. What is the patient's stage of eclampsia?

Прееклампсія Preeclampsia

Передсудомний період Pretrial period

Період тонічних судом Period of tonic seizures

Позасудомний період Non-seizure period

Період розрішення нападу Period of attack resolution

33 / 200
У дитини 9-ти років, яка хворіє на цукровий діабет, на тлі кишкової інфекції виникли і наростають неврологічні симптоми: порушення орієнтації, галюцинації, фокальні судоми. Наявні ознаки дегідратації III ступеня, дихальних порушень немає, запах ацетону відсутній. Який попередній діагноз? A 9-year-old child with diabetes developed and is increasing neurological symptoms against the background of an intestinal infection: disorientation, hallucinations, focal convulsions. Present signs dehydration of the III degree, there are no respiratory disorders, there is no smell of acetone. What is the previous diagnosis?

Діабетична кетоацидотична кома Diabetic ketoacidotic coma

Гіпоглікемічна кома Hypoglycemic coma

Енцефаліт Encephalitis

Пухлина головного мозку Brain tumor

Гіперосмолярна кома Hyperosmolar coma

34 / 200
У дитини 8-ми років підвищення температури до 39°С, нежить зі значними гнійними виділеннями, вологий кашель, кон’юнктивіт з гнійними виділеннями, світлобоязнь. На слизовій оболонці щік білуваті ділянки висівкоподібно злущеного епітелію. Який найбільш імовірний діагноз? An 8-year-old child has a temperature rise to 39°C, runny nose with significant purulent discharge, wet cough, conjunctivitis with purulent discharge, photophobia. On the mucous membrane the membranes of the cheeks are whitish areas of bran-like desquamated epithelium. What is the most likely diagnosis?

Корова краснуха Cow rubella

Висипний тиф Typhoid

Грип Flu

Кір Measles

Скарлатина Scarlatina

35 / 200
У дитини 4-тижневого віку через 2 тижні від початку захворювання, що проявляється блюванням ’’фонтаном’,’ вираженою гіпотрофією і ексикозом, педіатр припустив вроджений пілоростеноз. При надходженні до дитячого відділення стан вкрай важкий: риси обличчя загострені, адинамія, сильна спрага, анурія, запах сечі з рота, сірість шкірних покривів. У сироватці крові: натрій -135 моль/л, калій - 3,7 ммоль/л. Коматозний стан якого характеру розвинувся у дитини? In a 4-week-old child, 2 weeks after the onset of the disease, manifested by 'fountain' vomiting, pronounced hypotrophy and exicosis, the pediatrician suspected congenital pylorus stenosis. upon admission to the children's department, the condition is extremely severe: facial features are aggravated, adynamia, strong thirst, anuria, the smell of urine from the mouth, gray skin. In blood serum: sodium - 135 mol/l, potassium - 3.7 mmol/l. Comatose state what kind of character has the child developed?

Гіперосмолярна кома Hyperosmolar coma

Азотемічна кома Azotemic coma

Гіпоглікемічна кома Hypoglycemic coma

Гіпохлоремічна кома Hypochloremic coma

Кетоацидотична кома Ketoacidotic coma

36 / 200
Хворого 22-х років впродовж 2-х тижнів турбують ниючі болі в епігастральній ділянці оперізуючого характеру, нудота, за останні 2 дні - неодноразове блювання, що не приносить полегшення. Відмічає затримку газів. Об’єктивно: Ps- 92/хв., шкірні покрови ціанотичні, живіт здутий. При пальпації напруження м’язів немає. Болі локалізуються у ямці під грудьми і лівому реберно-хребтовому куті. Який попередній діагноз? A 22-year-old patient has been bothered by aching pains in the epigastric area of ​​a striated nature for 2 weeks, nausea, for the past 2 days - repeated vomiting that does not bring relief . Notes the retention of gases. Objectively: Ps- 92/min., the skin is cyanotic, the abdomen is distended. During palpation, there is no muscle tension. The pain is localized in the fossa under the chest and the left costo-spinal angle. What is the previous diagnosis?

Перфоративна виразка шлунка Perforative gastric ulcer

Гострий апендицит Acute appendicitis

Гострий панкреатит Acute pancreatitis

Гострий холецистит Acute cholecystitis

Хронічний гастрит Chronic gastritis

37 / 200
Першовагітна 25-ти років з терміном ге-стації 26 тижнів звернулася до лікаря жіночої консультації зі скаргами на погіршення загального стану, спрагу, слабкість, блювання, сонливість, болі по всьому животу впродовж останніх 8-ми діб. Хворіє на цукровий діабет. Запах ацетону з рота. Ps-100/хв., AT- 100/60 мм рт.ст. Рівень цукру в крові -19,4 ммоль/л. Набряки ніжних кінцівок. Який найбільш імовірний діагноз? A 25-year-old first-time pregnant woman with a gestation period of 26 weeks turned to the doctor of the women's consultation with complaints of worsening general condition, thirst, weakness, vomiting, drowsiness, pain all over the abdomen for the last 8 days. He has diabetes. The smell of acetone from the mouth. Ps-100/min., AT- 100/60 mm Hg. Blood sugar level -19.4 mmol/l. Edema tender limbs. What is the most likely diagnosis?

Кетонемічна кома Ketonemic coma

Гіпоглікемічна кома Hypoglycemic coma

Прееклампсія, тяжка форма Preeclampsia, severe form

Гіперлактацидемічна кома Hyperlactacidemic coma

Гіперосмолярна кома Hyperosmolar coma

38 / 200
Пацієнт 54-х років, що перебуває в стаціонарі з приводу гострого інфаркту міокарда, раптово знепритомнів. В ході огляду діагностовано зупинку серцевої діяльності та дихання, розпочато легенево-серцеву реанімацію. На ЕКГ спостерігається асистолія. Далі необхідно: A 54-year-old patient, who is in the hospital due to an acute myocardial infarction, suddenly fainted. During the examination, cardiac and respiratory arrest was diagnosed, cardiopulmonary resuscitation was started resuscitation. Asystole is observed on the ECG. Next, it is necessary:

Ввести внутрішньовенно строфантин Enter strophanthin intravenously

Дефібриляція Defibrillation

Ввести внутрішньосерцево адреналін Inject adrenaline intracardiacally

Ввести внутрішньовенно новокаїнамід Enter intravenous novocaine

Ввести внутрішньовенно лідокаїн Inject lidocaine intravenously

39 / 200
Хворий 65-ти років скаржиться на зміну кольору сечі. Сеча червона, з домішками кров’яних згустків, що нагадують 'черв’яків', болю немає, непокоїть слабкість. У сечі: гематурія. У крові: НЬ- 84 г/л, ШОЕ- 31 мм/год. Який найбільш імовірний діагноз? A 65-year-old patient complains of a change in the color of urine. The urine is red, with admixtures of blood clots that resemble 'worms', there is no pain, he is worried weakness. In the urine: hematuria. In the blood: Hb - 84 g/l, ESR - 31 mm/h. What is the most likely diagnosis?

Пухлина нирки Kidney tumor

Гострий пієлонефрит Acute pyelonephritis

Сечокам’яна хвороба Urolithiasis

Гострий гломерулонефрит Acute glomerulonephritis

Гострий цистит Acute cystitis

40 / 200
У хворого 46-ти років, що страждає на цукровий діабет II типу з загостренням IXС, розвинулася гіперосмолярна діабетична кома. Особливості гіперосмолярної коми: A 46-year-old patient suffering from type II diabetes with exacerbation of IXC developed hyperosmolar diabetic coma. Features of hyperosmolar coma:

Відсутність кетоацидозу No ketoacidosis

Часта олігурія Frequent oliguria

Наявність кетоацидозу Presence of ketoacidosis

Часта азотемія Frequent azotemia

Дегідратація Dehydration

41 / 200
Хворого 45-ти років виписано із ендокринологічного відділення після перенесеної гострої надниркової недостатності. Додаткові обстеження: рівень кортизону 80 мкг у 100 мл плазми, коефіцієнт Na+/K+ -32. На ЕКГ - ритм сину совий, дифузне зниження трофіки міокарду. Які профілактичні засоби необхідно рекомендувати хворому? A 45-year-old patient was discharged from the endocrinology department after suffering acute adrenal insufficiency. Additional examinations: cortisone level 80 μg in 100 ml of plasma, Na+/K+ ratio -32 . On the ECG - the son's rhythm is irregular, a diffuse decrease in the trophic state of the myocardium. What preventive measures should be recommended to the patient?

Бальнеотерапія Balneotherapy

Фізіотерапевтичне лікування Physiotherapy treatment

Хірургічне лікування Surgical treatment

Диспансерний нагляд з подальшою корекцією терапії Dispensary supervision with subsequent correction of therapy

Санаторно-курортне лікування Sanatorium-resort treatment

42 / 200
Дитині 6-ти років, оперованій з приводу розлитого перитоніту, в процесі забезпечення анестезіологічного засобу, проводиться штучна вентиляція легень у режимі помірної гіпервентиляції. На яких цифрах необхідно підтримувати рівень рС02 у КЛС-грамі? A 6-year-old child operated on for spilled peritonitis, in the process of providing an anesthetic agent, artificial lung ventilation is performed in the mode of moderate hyperventilation. At what numbers should the level be maintained pС02 in KLS-gram?

20-25 мм рт.ст. 20-25 mm Hg

30-35 мм рт.ст. 30-35 mm Hg

50-55 мм рт.ст. 50-55 mm Hg

35-40 мм рт.ст. 35-40 mm Hg

40-45 мм рт.ст. 40-45 mm Hg

43 / 200
Дитина 1-го року доставлена до лікарні з діагнозом: гостра ниркова недостатність, стадія олігоанурії. У хворого спостерігаються м’язова слабкість, аритмія, зміни на ЕКГ ( розширення інтервалів PQ та QRS, високі, вузькі та симетричні зубці Т). Які біохімічні зміни викликають ці порушення? A 1-year-old child was brought to the hospital with a diagnosis of acute renal failure, the stage of oligoanuria. The patient has muscle weakness, arrhythmia, changes in the ECG (extension PQ and QRS intervals, tall, narrow and symmetrical T-waves). What biochemical changes cause these disturbances?

Гіпокальціємія Hypocalcemia

Гіперкальціємія Hypercalcemia

Гіперкаліємія Hyperkalemia

Гіпокаліємія Hypokalemia

Ацидоз Acidosis

44 / 200
Хвора 60-ти років доставлена до приймального відділення непритомною, з гіпорефлексією, зниженим артеріальним тиском, брадикардією, брадипное. Рік тому перенесла субтотальну резекцію щитоподібної залози. Замісну терапію не проходила. Для якого стану характерні наведені ознаки? A 60-year-old patient was brought to the emergency room unconscious, with hyporeflexia, low blood pressure, bradycardia, bradypnea. A year ago, she underwent subtotal resection of the thyroid gland. Replacement therapy is not passed. For which condition are the given symptoms typical?

Мікседематозна кома Myxedema coma

Отруєння невідомою отрутою Poisoning with an unknown poison

Геморагічний шок Hemorrhagic shock

Гіпопаратиреоїдизм Hypoparathyroidism

Тиреотоксичний криз Thyrotoxic crisis

45 / 200
У дівчини 19-ти років після зґвалтування виникли постійна плаксивість, тривога, настрій став пригніченим, знизилася працездатність, рухи загальмовані, з’явилися думки про недоцільність життя. Визначте психопатологічний стан: After the rape, a 19-year-old girl developed constant tearfulness, anxiety, her mood became depressed, her work capacity decreased, her movements were inhibited, and thoughts about the futility of life appeared. Define psychopathological condition:

Реактивна депресія Reactive Depression

Тривожна депресія Anxious Depression

Судинна депресія Vascular depression

Ендогенна депресія Endogenous depression

Інволюційна депресія Involutional depression

46 / 200
Хвора 46-ти років скаржиться на появу протягом двох тижнів випорожнень кров’ю більш за 12 разів на добу, болю в животі, суглобах; болючі висипки у порожнині рота та нижніх кінцівках, набряк колінних та гомілковостопних суглобів, слабкість, підвищення температури тіла понад 39°С, втрату ваги тіла. Який метод обстеження є найбільш інформативним? A 46-year-old patient complains of the appearance of bloody stools more than 12 times a day, pain in the abdomen, joints, painful rashes in the oral cavity for two weeks and lower limbs, swelling of knee and ankle joints, weakness, increase in body temperature over 39°C, loss of body weight. Which examination method is the most informative?

Рентгенологічне дослідження кишечника X-ray examination of intestines

Біопсія кишечника та шкіри Intestinal and skin biopsy

Рентгенографія суглобів X-ray of joints

Ендоскопічне дослідження Endoscopy examination

Загальний аналіз крові General blood test

47 / 200
Пацієнтка 49-ти років скаржиться на стискаючий біль в голові, відчуття 'забитого цвяха', який посилюється наприкінці робочого дня. Об’єктивно: плаксива, знервована. Шкірні покриви бліді, гіпергідроз. Є тремор пальців рук, рівномірно підвищені сухожилкові рефлекси. Ps- 84/хв., АТ-140/80 мм рт.ст. Серце і легені в нормі. Надайте невідкладну допомогу як лікар швидкої допомоги: A 49-year-old female patient complains of a squeezing pain in her head, a feeling of a 'nail driven in', which intensifies at the end of the working day. Objectively: tearful, nervous. Skin the integuments are pale, hyperhidrosis. There is a tremor of the fingers, uniformly increased tendon reflexes. Ps- 84/min., BP-140/80 mm Hg. Heart and lungs are normal. Provide urgent care as an emergency physician:

Димедрол Diphenhydramine

Спазган Spazgan

Сибазон Sibazone

Анальгін Analgin

Настоянка валеріани Valerian Tincture

48 / 200
Під час діагностичної ендоскопії у хворого 45-ти років виникла асистолія, яка діагностована електрокардіографічно. З яких заходів слід негайно розпочати лікування? During a diagnostic endoscopy, a 45-year-old patient developed asystole, which was diagnosed electrocardiographically. What measures should be taken to treat it immediately?

Електрокардіостимуляція Electronic cardiostimulation

Інфузія гідрокарбонату натрію Sodium bicarbonate infusion

Дефібриляція Defibrillation

Масаж серця, ШВЛ, адреналін Heart massage, ventilator, adrenaline

Введення атропіну Atropine administration

49 / 200
У дитини 10-ти років зупинена артеріальна кровотеча (плечова артерія) через 10 хвилин після травми. При надходженні AT- 90/50 мм рт.ст., еритроцити - 2,5•1012/л, НЬ- 60 г/л. Раніше при переливанні цільної крові була анафілактоїдна реакція. Який препарат крові найбільш безпечний для корекції крововтрати? A 10-year-old child has stopped arterial bleeding (brachial artery) 10 minutes after the injury. At the arrival of AT- 90/50 mm Hg, erythrocytes - 2.5•1012/l, Hb- 60 g/l. Previously, there was an anaphylactoid reaction during transfusion of whole blood. Which blood preparation is the safest for blood loss correction?

Цільна кров Whole blood

Еритроцитарна маса Erythrocyte mass

Відмиті еритроцити Washed erythrocytes

Лейкоконцентрат Leukoconcentrate

Свіжозаморожена плазма Fresh-frozen plasma

50 / 200
Хворий 66-ти років скаржиться на серцебиття, біль у серці, слабкість. Симптоми з’явилися раптово, після фізичного навантаження. На ЕКГ: комплекси QRS поширені до 0,12 секунд, зубець Р перед шлуночковим комплексом відсутній, ЧСС- 200/хв. Який лікарський засіб є препаратом вибору в цьому випадку? A 66-year-old patient complains of palpitations, heart pain, weakness. The symptoms appeared suddenly, after physical exertion. On the ECG: QRS complexes spread to 0 ,12 seconds, the P wave before the ventricular complex is absent, the heart rate is 200/min. What drug is the drug of choice in this case?

Дигоксин Digoxin

Хінідин Quinidine

Обзидан Obzydan

Лідокаїн Lidocaine

Верапаміл Verapamil

51 / 200
Хворий 52-х років після ДТП від медичної допомоги відмовився, а через деякий час відчув зростаючу слабкість, запаморочення, тупий біль у лівому підребер’ї. Об’єктивно: загальмований, ЧСС- 130/хв., АТ-80/50 мм рт.ст. В анамнезі ІХС, гіпертонічна хвороба. Який імовірний діагноз? A 52-year-old patient refused medical help after a road accident, and after some time felt increasing weakness, dizziness, dull pain in the left hypochondrium. Objectively : inhibited, heart rate - 130/min., blood pressure - 80/50 mm Hg. History of coronary heart disease, hypertension. What is the probable diagnosis?

Інфаркт міокарда Myocardial infarction

Колапс на фоні внутрішньої кровотечі Collapse on the background of internal bleeding

Наслідки перенесеного стресу Consequences of stress

Напад стенокардії Attack of angina pectoris

Загострення хронічного панкреатиту Exacerbation of chronic pancreatitis

52 / 200
Хворий 45-ти років поступив у відділення зі скаргами на пекучий біль, сильне розпирання тканин пальців обох стоп. Об’єктивно: пухирі, наповнені геморагічною рідиною; після зняття пухирів видно некротизовану шкіру, котра нечутлива і неболюча. Який найбільш імовірний діагноз? A 45-year-old patient was admitted to the department with complaints of burning pain, severe swelling of the tissues of the fingers of both feet. Objectively: blisters filled with hemorrhagic fluid; after removal blisters show necrotic skin, which is insensitive and painless. What is the most likely diagnosis?

Відмороження II ступеня Defrosting of the II degree

Відмороження III ступеня Defrosting of the III degree

Термічний опік II ступеня Thermal burn of the second degree

Відмороження І ступеня Defrosting of the 1st degree

Термічний опік III А ступеня Thermal burn III A degree

53 / 200
У хворого із вкрай важким ступенем кістковомозкової форми гострої променевої хвороби у початковому періоді з’явилися: лихоманка, сильний головний біль, адинамія, біль у м’язах і суглобах. Який синдром гострої променевої хвороби у хворого? A patient with an extremely severe bone marrow form of acute radiation sickness in the initial period developed: fever, severe headache, adynamia, pain in muscles and joints What is the patient's syndrome of acute radiation sickness?

Гастроінтестинальний синдром Gastrointestinal syndrome

Геморагічний синдром Hemorrhagic syndrome

Синдром ураження нервової системи Syndrome of damage to the nervous system

Синдром інфекційних ускладнень Syndrome of infectious complications

Синдром загальної токсемії Syndrome of general toxemia

54 / 200
Лікар викликаний додому до жінки 28-ми років. Вагітності не запобігає. В анамнезі: 2 самовільні викидня у малому терміні, пологів не було. Висловлює скарги на затримку місячних 12 днів і появу кров’янистих виділень із статевих шляхів, тягнучі болі внизу живота і попереку. Яка тактика ведення? The doctor was called to the home of a 28-year-old woman. It does not prevent pregnancy. History: 2 spontaneous miscarriages in a short period of time, there was no childbirth. She complains about delayed menstruation 12 days and the appearance of bloody discharge from the genital tract, pulling pains in the lower abdomen and lower back. What are the management tactics?

Проведення експрес-тесту для підтвердження вагітності Conducting an express test to confirm pregnancy

Направлення у жіночу консультацію Referral to women's consultation

Екстрена госпіталізація у гінекологічне відділення Emergency hospitalization in the gynecological department

Направлення на планову госпіталізацію Referral for scheduled hospitalization

Призначення гемостатичної і спазмолітичної терапії, контроль підкладної Prescription of hemostatic and antispasmodic therapy, control of suppository

55 / 200
Хвора 54-х років з міомою матки й анемією (НЬ- 52 г/л, Ht- 17%) скаржиться на головний біль, слабкість, запаморочення. Лікар призначив жінці трансфузію одногрупних відмитих еритроцитів. Яка основна мета гемотрансфузії в цьому випадку? A 54-year-old female patient with uterine fibroids and anemia (НЭ- 52 g/l, Ht- 17%) complains of headache, weakness, dizziness. The doctor prescribed a transfusion of the same group of washed erythrocytes to the woman. What is the main purpose of hemotransfusion in this case?

Замісна Replaceable

Стимулювальна Stimulating

Поживна Nutrient

Гемодинамічна Hemodynamic

Детоксикаційна Detox

56 / 200
У хворої 24-х років захворювання почалося гостро з появи нудоти, багаторазового блювання, ознобу, болю у верхніх відділах живота. Практично одночасно з’явилися рідкі водянисті випорожнення без патологічних домішок 2 рази. Об’єктивно: стан середньої важкості, температура - 37,6°С, Ps-86/хв., AT-100/60 мм рт.ст., шкіра бліда, тургор збережений, язик незначно обкладений білим нальотом, живіт м’який, чутливий в епігастрії. Яку терапію слід провести у першу чергу? In a 24-year-old patient, the disease began acutely with the onset of nausea, repeated vomiting, chills, pain in the upper abdomen. Almost simultaneously, liquid watery stools appeared without pathological impurities 2 times. Objectively: a condition of moderate severity, temperature - 37.6°С, Ps-86/min., AT-100/60 mm Hg, skin is pale, turgor is preserved, the tongue is slightly coated with a white coating , the abdomen is soft, sensitive in the epigastrium. What therapy should be carried out in the first place?

Призначення сольового послаблюючого Prescription of saline laxative

Регідрон Regidron

5% розчин глюкози 5% glucose solution

Лоперамід Loperamide

Промивання шлунка Gastric lavage

57 / 200
Через 3 дні після чисельних укусів бліх у хворого різко підвищилась температура тіла до З9°С, з’явився сильний головний біль, біль у м’язах. Об’єктивно: гіперемія обличчя, ін’єкція судин склер, тремор язика, тахікардія, зниження AT. Мова невиразна. На другу добу в пахвовій ділянці утворився різко болючий під час пальпації бубон, шкіра над ним гаряча, напружена, незначний набряк навколишніх тканин. Яке захворювання найімовірніше? 3 days after numerous flea bites, the patient's body temperature rose sharply to 39°C, severe headache, muscle pain appeared. Ob' objectively: hyperemia of the face, injection of scleral vessels, tremor of the tongue, tachycardia, decrease in AT. Speech is unclear. On the second day, a drum was formed in the axillary area, which is painful during palpation, the skin above it is hot, tense, slight swelling of the surrounding tissues. What disease most likely?

Ботулізм Botulism

Бубонна форма чуми Bubonic form of plague

Бубонна форма туляремії Bubonic form of tularemia

Гострий лімфаденіт Acute lymphadenitis

Лімфогранулематоз Lymphogranulomatosis

58 / 200
В клініку дитячої хірургії надійшов хлопчик 6-ти місяців у важкому стані через 36 годин після початку захворювання з клінікою інвагінації кишечнику. В сім’ї у дитини дідусь та батько хворіють на гемофілію. Дитині показане негайне оперативне втручання. Яка передопераційна підготовка повинна проводитися хворому? A 6-month-old boy was admitted to the children's surgery clinic in critical condition 36 hours after the onset of the disease with intestinal intussusception. The child's family includes a grandfather and a father suffer from hemophilia. Immediate surgical intervention is indicated for the child. What preoperative preparation should be carried out for the patient?

Переливання препаратів крові та криопреципітату Transfusion of blood products and cryoprecipitate

Інфузійна терапія сольовими розчинами Infusion therapy with saline solutions

Інфузійна терапія з використанням колоїдних розчинів Infusion therapy using colloid solutions

Переливання плазми після оперативного втручання Plasma transfusion after surgery

Гемостатична терапія під час втручання Hemostatic therapy during intervention

59 / 200
Хворий 38-ми років, що страждає на нирковий туберкульоз, в непритомному стані привезений родичами до районної лікарні. Об’єктивно: кома 1. Гіпотрофія 2 ст. Шкіра та слизові оболонки коричневого кольору з бронзовим полиском. На шкірі спостерігаються окремі осередки вітиліго. AT- 65/0 мм рт.ст., ЧСС- 44/хв. Діагностована гостра надниркова недостатність. Який препарат треба ввести в першу чергу? A 38-year-old patient suffering from renal tuberculosis was brought by relatives to the district hospital in an unconscious state. Objectively: coma 1. Hypotrophy 2nd stage. Skin and brown mucous membranes with a bronze sheen. Individual foci of vitiligo are observed on the skin. AT- 65/0 mmHg, heart rate- 44/min. Acute adrenal insufficiency is diagnosed. What drug should be administered first?

Мезатон Mesaton

Дексаметазон Dexamethasone

Новодрин Novodrin

Адреналін Adrenaline

Реополіглюкін Rheopoliglyukin

60 / 200
В медичну роту механізованої бригади поступили з поля бою 10 легкопоранених, 15 поранених середньої важкості і 20 тяжкопоранених. Якими принципами та вимогами керується медичний персонал, який виконує сортування поранених в приймальносортувальному відділенні медичної роти? 10 lightly wounded, 15 moderately wounded, and 20 seriously wounded entered the medical company of the mechanized brigade from the battlefield. What are the principles and requirements of the medical personnel who triage the wounded in reception and sorting department of the medical company?

За потребою місця надання медичної допомоги пораненим As required by the location of providing medical assistance to the wounded

За потребою якнайшвидшої евакуації поранених до наступного етапу медичної евакуації For the need of the earliest possible evacuation of the wounded to the next stage of medical evacuation

Сортування поранених за принципом 'Пироговських рядів' Sorting of the wounded according to the principle of 'Pie rows'

Єдина класифікація хвороб і травм та єдині вимоги до методики сортування Unified classification of diseases and injuries and unified requirements for sorting methods

За потребою розподілу та по важкості стану поранених By the need for distribution and by the severity of the condition of the wounded

61 / 200
При променевому ураженні шкіри у хворого 33-х років спостерігається вологий дерматит, пухирі малі, ненапружені, займають менш ніж 50% поля. Така клінічна картина виникає при дозі опромінення: In case of radiation damage to the skin, a 33-year-old patient has moist dermatitis, the blisters are small, non-tense, occupying less than 50% of the field. This clinical picture occurs with a dose of radiation :

15-20 Гр 15-20 Gy

10-15 Гр 10-15 Gy

5-8 Гр 5-8 Gy

7-13 Гр 7-13 Gy

20-30 Гр 20-30 Gy

62 / 200
Вагітна 40 тижнів з бурхливою пологовою діяльністю, клінічно вузьким тазом. Під час індукції наркозу виникло блювання. Після інтубації проведена санація дихальних шляхів та ротової порожнини. В подальшому розвинувся ціаноз, підвищився центральний венозний тиск, над легенями різнокаліберні вологі хрипи, підвищення тиску на вдиху. При контролі газів крові - значна гіпоксемія. Яка причина цього стану? 40 weeks pregnant with violent labor, clinically narrow pelvis. Vomiting occurred during the induction of anesthesia. After intubation, sanitation of the respiratory tract and oral cavity was carried out. Subsequently, cyanosis developed , increased central venous pressure, moist rales of various caliber over the lungs, increased pressure on inspiration. When blood gases are controlled, there is significant hypoxemia. What is the cause of this condition?

Емболія навколоплідними водами Amniotic fluid embolism

Набряк легенів Pulmonary edema

Ендотоксичний шок Endotoxic shock

Синдром Мендельсона Mendelssohn syndrome

Інфаркт міокарда Myocardial infarction

63 / 200
Хлопчик 11-ти років госпіталізований до реанімаційного відділення у зв’язку з непритомністю на спортивному майданчику. В анамнезі - епізоди непритомностей, болі у грудях при фізичному навантаженні. Об’єктивно: пульс двогорбий, посилення верхівкового поштовху серця, вислуховується третій тон порушення серцевого ритму. ЕКГ: поліморфні шлуночкові екстра-систоли. ЕхоКС - асиметрична гипертрофія міжшлуночкової перегородки. Який шок має місце у дитини? An 11-year-old boy was hospitalized in the intensive care unit due to fainting on the sports field. He has episodes of fainting, chest pains during physical exertion. 'objectively: double-humped pulse, increased apical impulse of the heart, the third sound of heart rhythm disturbance is heard. ECG: polymorphic ventricular extra-systoles. EchoCS - asymmetric hypertrophy of the interventricular septum. What kind of shock is there in the child?

Кардіогенний шок Cardiogenic shock

Анафілактичний шок Anaphylactic shock

Травматичний шок Traumatic shock

Септичний шок Septic shock

Гіповолемічний шок Hypovolemic shock

64 / 200
Дитина 15-ти років доставлена непритомною до приймального відділення бригадою швидкої допомоги у супроводі батьків. Хворий потребує негайної допомоги і госпіталізації у ВРІТ. Чи потрібно лікарю отримати згоду на медичне втручання? A 15-year-old child was brought unconscious to the reception department by an ambulance team accompanied by his parents. The patient needs immediate help and hospitalization in the intensive care unit. Does the doctor need to obtain medical consent intervention?

Батьки можуть відмовитись від надання допомоги Parents can refuse assistance

В даному випадку згода не потрібна Consent is not required in this case

Лікар зобов’язаний пояснити пацієнтові в доступній формі методи і об’єм медичного втручання The doctor is obliged to explain to the patient in an accessible form the methods and scope of medical intervention

Батьки мають право вибору методів медичного втручання Parents have the right to choose methods of medical intervention

Медичне втручання особам від 15 до 18 років проводиться за їх згодою Medical intervention for persons aged 15 to 18 is carried out with their consent

65 / 200
Пацієнт під час ковтання цукерки захрипів, схопився за шию, посинів і через З хвилини знепритомнів. Об’єктивно: свідомість відсутня, обличчя і шия ціанотичні, нерегулярні та непродуктивні рухи грудної клітки. На сонних артеріях - слабка пульсація. При пальцевій ревізії ротоглотки визначається округле стороннє тіло, змістити яке не вдається. Спроба змістити його при перекиданні пацієнта через коліно - невдала. Які подальші дії? While swallowing a candy, the patient snored, grabbed his neck, turned blue and passed out after 3 minutes. Objectively: consciousness is absent, the face and neck are cyanotic, irregular and unproductive movements of the chest. There is a weak pulsation on the carotid arteries. During a finger inspection of the oropharynx, a rounded foreign body is identified, which cannot be dislodged. An attempt to dislodge it while rolling the patient over the knee is unsuccessful. What are the further actions?

Нижня трахеотомія Lower tracheotomy

Крікотиреотомія Cricothyrotomy

Пункція трахеї Trache puncture

Верхня трахеотомія Upper tracheotomy

Транспортування до стаціонару Transportation to hospital

66 / 200
Жінка 69-ти років раптово втратила свідомість, родичі хворої викликали швидку допомогу, яка прибула через 35 хвилин. При огляді: пульс відсутній на сонних артеріях, дихання відсутнє, зіниці розширені, на ЕКГ - асистолія. Які дії бригади? A 69-year-old woman suddenly lost consciousness, the patient's relatives called an ambulance, which arrived 35 minutes later. On examination: there was no pulse on the carotid arteries, no breathing, The pupils are dilated, asystole on the ECG. What are the actions of the team?

Негайне проведення серцево-легеневої реанімації Immediate cardiopulmonary resuscitation

Нанесення прекардіяльного удару Inflicting a precardial blow

Проведення дефібриляції Performing defibrillation

Серцево-легенева реанімація не показана Cardiopulmonary resuscitation is not shown

Внутрішньовенне введення лідокаїну Intravenous administration of lidocaine

67 / 200
У породіллі 16-ти років почалась гіпотонічна кровотеча у ранньому післяпологовому періоді. Об’єм крововтрати 1,6% від маси тіла, Ps- 115/хв., систолічний AT- 80 мм рт.ст., ЦВТ- 35 мм вод.ст. Який найбільш імовірний діагноз? A 16-year-old woman in labor started hypotonic bleeding in the early postpartum period. Blood loss volume 1.6% of body weight, Ps- 115/min., systolic AT - 80 mm Hg, CVT - 35 mm Hg. What is the most likely diagnosis?

Гіпотонічна кровотеча у ранньому післяпологовому періоді. Геморагічний шок II ступеня Hypotonic bleeding in the early postpartum period. Hemorrhagic shock of the II degree

Гіпотонічна кровотеча у ранньому післяпологовому періоді. Геморагічний шок І ступеня Hypotonic bleeding in the early postpartum period. Hemorrhagic shock of the 1st degree

Гіпотонічна кровотеча у ранньому післяпологовому періоді без ознак геморагічного шоку Hypotonic bleeding in the early postpartum period without signs of hemorrhagic shock

Гіпотонічна кровотеча у ранньому післяпологовому періоді. Геморагічний шок IV ступеня Hypotonic bleeding in the early postpartum period. IV degree hemorrhagic shock

Гіпотонічна кровотеча у ранньому післяпологовому періоді. Геморагічний шок III ступеня Hypotonic bleeding in the early postpartum period. III degree hemorrhagic shock

68 / 200
У хворого 37-ми років, що вживав напередодні копчену рибу власного виробництва, поступово розвинулись м’язова слабкість, неможливість читати газетний текст, на другий день - порушення ковтання. Об’єктивно: стан тяжкий, ціаноз шкіри, страбізм, ЧД- 50/хв., Ps- 120/хв. Іолос гугнявий, парез зіниць, метеоризм 2 ступеня. Випорожнення відсутні 2 доби. Який лікувальний захід показаний у першу чергу? A 37-year-old patient, who had consumed smoked fish of his own production the day before, gradually developed muscle weakness, the inability to read a newspaper text, and on the second day - a swallowing disorder . Objectively: the condition is severe, cyanosis of the skin, strabismus, BH - 50/min., Ps - 120/min. Iolos hugnavy, paresis of the pupils, flatulence of the 2nd degree. There are no bowel movements for 2 days. What medical measure is indicated in the first place?

Штучне дихання 'рот у рот' Mouth-to-mouth artificial respiration

Введення діуретиків Introduction of diuretics

Трахеостомія Tracheostomy

Штучна вентиляція легень Artificial lung ventilation

Призначення прозерину Destination of proserin

69 / 200
Хворий 40-ка років скаржиться на інтенсивні болі в горлі при ковтанні, виражену задишку, що посилюється у горизонтальному положенні і супроводжується нападами задухи. При непрямій ларингоскопії виявлений абсцес надгортанника. Відділи гортані, розташовані нижче, оглянути не вдається. Від трахеотомії хворий категорично відмовився. На висоті чергового нападу задухи розвинулися ціаноз, судоми та наступила зупинка дихання. Яку першочергову допомогу слід надати? A 40-year-old patient complains of intense pain in the throat when swallowing, pronounced shortness of breath, which worsens in a horizontal position and is accompanied by attacks of suffocation. During indirect laryngoscopy, an epiglottis abscess was detected . It is not possible to examine the departments of the larynx located below. The patient categorically refused a tracheotomy. At the height of another attack of suffocation, cyanosis, convulsions and respiratory arrest occurred. What first aid should be provided?

Ендоларингеальне розкриття абсцесу під контролем прямої ларингоскопії Endolaryngeal abscess opening under the control of direct laryngoscopy

Трахеотомія Tracheotomy

Конікотомія Conicotomy

Інтубація трахеї Tracheal intubation

Штучна вентиляція легенів методом з рота в рот Artificial lung ventilation by the mouth-to-mouth method

70 / 200
У хворої 59-ти років раптово з’явився сильний біль у епігастрії, непритомність. Останні роки часті загострення виразкової хвороби. Яке ускладнення є найбільш імовірним? A 59-year-old patient suddenly developed severe pain in the epigastrium, fainting. In recent years, frequent exacerbations of peptic ulcer disease. What complication is most likely?

Прободіння, як ускладнення виразкової хвороби Woke up as a complication of peptic ulcer disease

Розшаровуюча аневризма аорти Dissecting aortic aneurysm

Кровотеча, як ускладнення виразкової хвороби Bleeding as a complication of peptic ulcer

Гострий холецистит на тлі виразкової хвороби Acute cholecystitis on the background of peptic ulcer disease

Гострий інфаркт міокарда Acute myocardial infarction

71 / 200
У хлопчика 15-ти років раптово підвищилася температура тіла до 39,8°С, виник біль у горлі під час ковтання. На другу добу на обличчі, шиї, верхній частині тулуба на тлі гіперемованої шкіри тулуба виник дрібнокрапковий розеольозний висип з блідим носогубним трикутником. Який найбільш імовірний діагноз? A 15-year-old boy had a sudden rise in body temperature to 39.8°C, a sore throat while swallowing. On the second day, his face, neck, on the upper part of the body, on the background of hyperemic skin of the body, a small dot roseolosis rash with a pale nasolabial triangle appeared. What is the most likely diagnosis?

Кір Measles

Інфекційний мононуклеоз Infectious mononucleosis

Вітряна віспа Chicken Pox

Скарлатина Scarlatina

Краснуха Krasnukha

72 / 200
У хворого 44-х років скарги на загальну слабкість, гіпертермію до З9°С, свербіння шкіри, висип, болісність і припухлість суглобів. Об’єктивно: AT- 70/40 мм рт.ст., папульозномакульозний висип, генералізоване збільшення лімфовузлів, припухлість суглобів, збільшення селезінки. 7 днів тому у травмпункті провели первинну хірургічну обробку рани стопи та ввели протиправцеву сироватку. Вкажіть причину розвитку даного стану: A 44-year-old patient complains of general weakness, hyperthermia up to 39°C, skin itching, rash, joint pain and swelling. Objectively: AT- 70/40 mmHg, papulo-macular rash, generalized enlargement of lymph nodes, swelling of joints, enlargement of the spleen. 7 days ago, primary surgical treatment of the foot wound was performed in the trauma center and anti-tetanus serum was administered. Specify the reason for the development of this condition:

Анафілактична реакція Anaphylactic reaction

Сироваткова хвороба Serum sickness

Менінгіт Meningitis

Сепсис Sepsis

Геморагічний васкуліт Hemorrhagic vasculitis

73 / 200
У потерпілого 36-ти років, що переніс закриту черепно-мозкову травму, в процесі проведення ехо-енцефалоскопії виявлено зміщення серединного еха на 5 мм. Про що це свідчить? In a 36-year-old victim who suffered a closed craniocerebral injury, during the echo-encephaloscopy, a 5 mm shift of the median echo was found. What does this indicate ?

Травматичний субарахноїдальний крововилив Traumatic subarachnoid hemorrhage

Перелом основи черепа Skull base fracture

Ехо-ЕС патології не виявила Echo-ES did not detect pathology

Тріщина кісток склепіння черепа Crack of the bones of the vault of the skull

Наявність підоболонкової гематоми Presence of submucosal hematoma

74 / 200
У 34 тижні вагітності в жінки з хронічною артеріальною гіпертензією середнього ступеня тяжкості, плацентарною недостатністю, затримкою розвитку плоду III ступеня, з’явився раптовий біль у матці. Об’єктивно: Ps- 120/хв., AT- 70/40 мм рт.ст. Матка в гіпертонусі. Серцебиття плоду - відсутнє. Із піхви помірні кров’янисті виділення. Яка причина антенатальної загибелі плоду? In the 34th week of pregnancy, a woman with chronic arterial hypertension of moderate severity, placental insufficiency, fetal growth retardation of the III degree, developed sudden pain in the uterus. Ob' objectively: Ps- 120/min., AT- 70/40 mm Hg. The uterus is hypertonic. The fetal heartbeat is absent. There is moderate bloody discharge from the vagina. What is the cause of antenatal death of the fetus?

Передчасне відшарування плаценти, легка форма Premature placental abruption, mild form

Затримка внутрішньоутробного розвитку Intrauterine developmental delay

Гіпертонічна хвороба Hypertensive disease

Передчасне відшарування плаценти, тяжка форма Premature placental abruption, severe form

Плацентарна недостатність Placental insufficiency

75 / 200
У 15-річного хлопчика діагностовано бубонну форму чуми. Які антибактеріальні препарати слід призначити дитині? A 15-year-old boy was diagnosed with bubonic plague. What antibacterial drugs should be prescribed to the child?

Бісептол або сульфален Biseptol or sulfalen

Пеніцилін або цефотаксим Penicillin or cefotaxime

Стрептоміцин або хлорамфенікол Streptomycin or Chloramphenicol

Тетрациклін або доксициклін Tetracycline or doxycycline

Гентаміцин або нетроміцин Gentamicin or Netromycin

76 / 200
Хворий 43-х років викликав швидку медичну допомогу. Скаржиться на різкий біль у лівій половині живота з іррадіацією в ліву ногу. В анамнезі сечокам’яна хвороба. Об’єктивно: шкіра бліда, Ps- 100/хв., АТ-160/90 мм рт.ст. Симптом Пастернацького різко позитивний зліва. Почати лікування найбільш доцільно із призначення: A 43-year-old patient called an ambulance. He complains of sharp pain in the left half of the abdomen with radiation to the left leg. He has a history of urolithiasis. Ob' objectively: the skin is pale, Ps-100/min., BP-160/90 mm Hg. Pasternacki's symptom is sharply positive on the left. It is most appropriate to start treatment with the appointment:

Морфін в/в Morphine IV

Баралгін в/в Baralgin v/v

Уролесан перорально Urolesan oral

Анальгін в/м Analgin v/m

Спазмалгон перорально Spazmalgon orally

77 / 200
У хворого з критичним станом контролюються показники газового складу артеріальної крові. Наявні ознаки дихального ацидозу. Які показники газового складу підтверджують дихальний ацидоз? In a patient with a critical condition, the parameters of the gas composition of the arterial blood are monitored. There are signs of respiratory acidosis. What indicators of the gas composition confirm respiratory acidosis?

Ізольоване зниження рС02 Isolated reduction of pС02

Підвищення pH і р02 Increasing pH and p02

Підвищення рС02 Increasing pС02

Підвищення pH і рС02 Increasing pH and pC02

Зниження pH і р02 Decrease in pH and p02

78 / 200
У спортсмена після підйому штанги з’явились інтенсивний біль в грудній клітці, задишка у спокої. Об’єктивно: при перкусії над лівою половиною грудної клітки - тимпаніт, голосове тремтіння відсутнє; при аускультації - різко ослаблене дихання. Над правою половиною - дихання везикулярне. Який попередній діагноз? After lifting a barbell, the athlete developed intense pain in the chest, shortness of breath at rest. Objectively: when percussion over the left half of the chest - tympanitis, vocal there is no tremor; during auscultation - sharply weakened breathing. Over the right half - vesicular breathing. What is the preliminary diagnosis?

Тромбоемболія легеневої артерії Thromboembolism of the pulmonary artery

Міжреберна невралгія Intercostal neuralgia

Крупозна пневмонія Croup pneumonia

Інфаркт міокарда Myocardial infarction

Лівобічний пневмоторакс Left sided pneumothorax

79 / 200
Лікаря викликали до хворого з черепно-мозковою травмою. Свідомість втрачена. Ціаноз шкірних покривів. Пульс на сонних артеріях не пальпується. Який першочерговий захід слід виконати? A doctor was called to a patient with a brain injury. He lost consciousness. Cyanosis of the skin. The pulse on the carotid arteries is not palpable. What is the first step to be taken?

Налагодження ЕКГ-контролю Setting up ECG control

Вивільнення дихальних шляхів, масаж серця та ІІГВЛ Airway release, heart massage and IIGVL

Виклик на консультацію нейрохірурга Call for neurosurgeon consultation

Проведення електричної кардіоверсії Performing electrical cardioversion

Проведення комп’ютерної томографії голови Computed tomography of the head

80 / 200
У новонародженої дитини на третю добу життя з’явилися ознаки шлунково-кишкової кровотечі (блювання з домішками крові, мелена), постгеморагічної анемії. З якого розчину слід розпочати парентеральне лікування геморагічного синдрому у дитини? On the third day of life, a newborn child showed signs of gastrointestinal bleeding (vomiting with blood impurities, melena), posthemorrhagic anemia. From which solution should parenteral treatment of hemorrhagic syndrome in a child?

Глюкозо-сольовий розчин Glucose-saline solution

Тромбоцитарна маса Platelet mass

Амінокапронова кислота Aminocaproic acid

Свіжозаморожена плазма Fresh-frozen plasma

Реополіглюкін Rheopoliglyukin

81 / 200
Бригаду швидкої допомоги викликано на місце пожежі, де було виявлено дитину 5-ти років з ознаками опіків на обличчі. Об’єктивно: диспное, захриплість голосу, стрідор. Перелом великогомілкової кістки. Ознаки наростаючої гіпоксемії з метаболічним ацидозом. Яка лікарська тактика? The ambulance crew was called to the scene of the fire, where a 5-year-old child was found with signs of burns on his face. Objectively: dyspnoea, hoarseness, stridor. Fracture of the tibia. Signs of increasing hypoxemia with metabolic acidosis. What are the medical tactics?

Термінова госпіталізація до ВРІТ Urgent hospitalization in the ICU

Спостереження дільничного педіатра Observation of the district pediatrician

Звертання в травмпункт Address to trauma center

Лікування в опіковому відділенні Treatment in the burn department

Амбулаторне лікування Outpatient treatment

82 / 200
Хлопчик 10-ти років знаходиться на лікуванні з приводу бронхіальної астми. Вранці хлопчик встав з ліжка і раптово з’явились загальна слабкість, запаморочення, блідість шкіри, холодний піт, шум у вухах, хворий впав. Об’єктивно: зіниці розширені, тони серця різко ослаблені, пульс ниткоподібний, ЧСС- 65/хв., AT- 60/40 мм рт.ст. Яке ускладнення розвинулось у хворого? A 10-year-old boy is being treated for bronchial asthma. In the morning, the boy got out of bed and suddenly developed general weakness, dizziness, pale skin, cold sweat , tinnitus, the patient fell. Objectively: the pupils are dilated, the heart sounds are sharply weakened, the pulse is filiform, heart rate - 65/min., AT - 60/40 mm Hg. What complication developed in the patient?

Інфекційно-токсичний шок Infectious-toxic shock

Кардіогенний шок Cardiogenic shock

Серцева недостатність Heart failure

Непритомність Fainting

Бактеріальний шок Bacterial shock

83 / 200
З вогнища радіаційної катастрофи до медичного пункту полку доставлено 15 потерпілих. Через 20-25 хвилин після вибуху потерпілі стали відчувати різку слабкість, запаморочення, нудоту, з’явилось невпинне блювання. Об’єктивно: стан хворих важкий. Кволі, апатичні, на запитання відповідають з великою затримкою. ЧСС- 120/хв. з частими екстрасистолами, AT- 70/30 мм рт.ст., ЧД- 28/хв. За даними індивідуального дозиметра доза отриманого випромінювання склала 8 Гр. Якій стадії гострої променевої хвороби відповідає наведена симптоматика? 15 victims were brought from the site of the radiation disaster to the regimental medical center. 20-25 minutes after the explosion, the victims began to feel sharp weakness, dizziness, nausea, there was an incessant vomiting. Objectively: the condition of the patients is severe. Weak, apathetic, they answer questions with a long delay. Heart rate - 120/min with frequent extrasystoles, AT - 70/30 mm Hg, BP - 28/min. According to the data of the individual dosimeter, the dose of received radiation was 8 Gy. What stage of acute radiation sickness corresponds to the given symptomatology?

IV IV

І And

V V

II II

III III

84 / 200
Дитина 5-ти років скаржиться на лихоманку, пронос, блювання та запаморочення. У крові: Ht- 0,55, Na+- 118 ммоль/л, К+- 5,9 ммоль/л. Які розлади водно-електролітного балансу спостерігаються? A 5-year-old child complains of fever, diarrhea, vomiting and dizziness. In the blood: Ht- 0.55, Na+- 118 mmol/l, K+ - 5.9 mmol/l. What disorders of the water-electrolyte balance are observed?

Гіперкаліємія Hyperkalemia

Гіпотонічна дегідратація, гіперкаліємія Hypotonic dehydration, hyperkalemia

Гіпотонічна дегідратація, гіпокаліємія Hypotonic dehydration, hypokalemia

Ізотонічна дегідратація, гіперкаліємія Isotonic dehydration, hyperkalemia

Розладів водно-електролітного балансу немає There are no water-electrolyte balance disorders

85 / 200
У дворічного хлопчика сплутана свідомість, невпинне блювання кров’ю, мелена, глибоке шумне дихання, печінка +6 см з-під краю реберної дуги. З’явились судоми. Стан погіршився декілька годин тому. В анамнезі - з приводу застуди впродовж трьох днів дитина отримувала по дві таблетки ліків на день. Який лікарський засіб міг викликати такі побічні явища? A two-year-old boy has confused consciousness, incessant vomiting of blood, grinding, deep noisy breathing, liver +6 cm from the edge of the costal arch. Convulsions appeared The condition worsened a few hours ago. In the anamnesis, the child received two tablets of medicine a day for a cold for three days. What medicine could cause such side effects?

Ампіцилін Ampicillin

Парацетамол Paracetamol

Аспірин Aspirin

Бісептол Biseptol

Анальгін Analgin

86 / 200
Пологи у 36 тижнів. Вагітність перебігала на тлі загрози переривання у 28 тижнів, гострої респіраторної вірусної інфекції у 24 тижні. Навколоплідні води світлі. Новонароджена дівчинка з масою 2400 г, довжиною 45 см. При народженні самостійне дихання відсутнє, м’язовий тонус значно знижений. Після проведення штучної вентиляції легенів за допомогою маски та мішка 100% киснем впродовж 30-ти секунд самостійне дихання не відновилося, частота серцевих скорочень складає 50/хв., зберігається акроціаноз. Яка подальша тактика лікаря? Delivery at 36 weeks. The pregnancy proceeded against the background of the threat of termination at 28 weeks, an acute respiratory viral infection at 24 weeks. The amniotic fluid is light. A newborn girl weighing 2400 g , 45 cm long. At birth, there is no independent breathing, muscle tone is significantly reduced. After performing artificial ventilation of the lungs using a mask and a bag with 100% oxygen for 30 seconds, independent breathing did not recover, the heart rate is 50/min. acrocyanosis persists. What is the doctor's further tactics?

Санація верхніх дихальних шляхів Upper respiratory tract rehabilitation

Введення адреналіну Adrenaline injection

Киснева допомога вільним потоком Free Flow Oxygen Support

Непрямий масаж серця Indirect heart massage

Корекція гіповолемії Correction of hypovolemia

87 / 200
У пацієнта з виразковою хворобою дванадцятипалої кишки в анамнезі скарги на біль, нудоту, блювання, відчуття розпирання в лівому підребер’ї. При рентгенологічному дослідженні виявлена затримка контрастної рідини у шлунку до 24 годин. Для якого ускладнення виразкової хвороби характерні дані клінічні прояви і рентгенологічна картина? A patient with a duodenal ulcer has a history of complaints of pain, nausea, vomiting, a feeling of distention in the left hypochondrium. X-ray examination revealed a retention of contrast fluid in stomach for up to 24 hours. For which complication of peptic ulcer disease are these clinical manifestations and X-ray picture typical?

Кровотеча Bleeding

Малігнізація виразки Ulcer malignancy

Стеноз воротаря Stenosis of the goalkeeper

Перфорація виразки Perforation ulcer

Пенетрація виразки Ulcer penetration

88 / 200
Хвора 25-ти років поступила в лікарню з ознаками ядухи, гавкаючим кашлем, ціанозом обличчя. При огляді порожнини рота встановлено набряк язика, м’якого піднебіння, мигдаликів. AT- 85/60 мм рт.ст. За 30 хвилин до того вживала полуниці. Який найбільш імовірний діагноз? A 25-year-old patient was admitted to the hospital with signs of dysentery, a barking cough, cyanosis of the face. An examination of the oral cavity revealed swelling of the tongue, soft palate, and tonsils. AT- 85/60 mm Hg. 30 minutes before that she used strawberries. What is the most likely diagnosis?

Набряк Квінке Quincke edema

Напад бронхіальної астми Bronchial asthma attack

Анафілактичний шок Anaphylactic shock

Набряк гортані Swelling of the larynx

Колапс Collapse

89 / 200
Хворий 38-ми років протягом тривалого часу відчував зубний біль. Згодом з’явилися біль голови і ока, загальна слабкість, підвищилася температура тіла. Об’єктивно: хемоз кон’юнктиви, застійна ін’єкція очного яблука, екзофтальм, обмеження рухів очного яблука. В крові: лейкоцитоз, ІПОЕ-24 мм/год. На рентгенограмі орбіт патології не виявлено. Який найбільш імовірний діагноз? A 38-year-old patient had a toothache for a long time. Later, headache and eye pain appeared, general weakness, body temperature rose. Objectively: chemosis conjunctivae, congestive injection of the eyeball, exophthalmos, restriction of eyeball movements. In the blood: leukocytosis, IPOE-24 mm/h. No pathology was detected on the x-ray of the orbits. What is the most likely diagnosis?

Ретробульбарный крововилив Retrobulbar hemorrhage

Флегмона орбіти Phlegmon of the orbit

Запальний псевдотумор орбіти Inflammatory pseudotumor of the orbit

Целюліт орбіти Cellulitis of the orbit

Теноніт Tenonite

90 / 200
Чоловік віком 47 років, працює рентгенологом впродовж 22-х років скаржиться на світлобоязнь, сльозотечу, зниження гостроти зору обох очей. 10 місяців тому переніс кон’юнктивіт обох очей. Гострота зору обох очей 0,7 Об’єктивно: легка світлобоязнь. Рогівка фарбується флюоресцеїном у вигляді окремих крапок. Чутливість рогівки знижена. Інші відділи ока не змінені. Який діагноз відповідає описаній клінічній картині? A 47-year-old man, who has been working as a radiologist for 22 years, complains of photophobia, tearing, decreased visual acuity in both eyes. 10 months ago, he suffered conjunctivitis in both eyes . Visual acuity of both eyes 0.7 Objectively: mild photophobia. The cornea is stained with fluorescein in the form of separate dots. The sensitivity of the cornea is reduced. Other parts of the eye are not changed. What diagnosis corresponds to the described clinical picture?

Помутніння рогівки Opacity of the cornea

Променеве ураження рогівки Corneal radiation damage

Помутніння склистого тіла Opacity of the vitreous body

Вікова катаракта Age cataract

Інфекційний кон’юнктивіт Infectious conjunctivitis

91 / 200
У доношеної дитини на 2 день життя діагностовано гемолітичну хворобу новонародженого по Rh-фактору, жовтянична форма. Через 10 години після народження з’явилась субіктеричність шкірних покровів. Розпочато фототерапію, рівень білірубіну сироватки крові - 160 мкмоль/л, через 4 години - 200 мкмоль/л. Яку тактику слід обрати? On the 2nd day of life, a full-term child was diagnosed with hemolytic disease of the newborn according to the Rh factor, jaundiced form. 10 hours after birth, subicteric skin appeared. Phototherapy was started , serum bilirubin level - 160 μmol/l, after 4 hours - 200 μmol/l. What tactics should be chosen?

Продовжити фототерапію Continue phototherapy

Операція замінного переливання крові Blood replacement operation

Інфузійна терапія Infusion therapy

Медикаментозна терапія Drug therapy

Спостереження Observations

92 / 200
Хворий 51-го року доставлений до відділення кардіореанімації (інтенсивної терапії), скаржиться на тривалий пекучий загрудинний біль, який виник 40 хвилин тому, задишку. Об’єктивно: в легенях - дихання з жорстким відтінком, тони серця різко ослаблені, AT-110/70 мм рт.ст. На ЕКГ виявлено підйом сегменту ST на 5 мм над ізолінією у відведеннях I, V1 — V4 та депресію сегмента ST у відведеннях III і aVF. Який препарат потрібно застосувати у даній ситуації? A 51-year-old patient was brought to the cardioresuscitation (intensive care) department, complaining of long-lasting burning chest pain that started 40 minutes ago, shortness of breath. Objectively: in the lungs - breathing with a harsh tone, heart sounds are sharply weakened, AT-110/70 mm Hg. The ECG revealed an elevation of the ST segment 5 mm above the isoline in leads I, V1 — V4 and depression of the ST segment in leads III and aVF What drug should be used in this situation?

Добутамін Dobutamine

Аміодарон Amiodarone

Метопролол Metoprolol

Альтеплаза Alteplase

Нітрогліцерин Nitroglycerin

93 / 200
Хвора 64-х років перебуває на лікуванні в кардіологічному відділенні з приводу гіпертонічної хвороби. На момент огляду AT- 200/110 мм рт.ст., ЧСС- 96/хв. Застосування якого препарату ПРОТИПОКАЗАНО в даній ситуації? A 64-year-old patient is being treated in the cardiology department for hypertension. At the time of examination, AT- 200/110 mm Hg, heart rate- 96 /min. The use of which drug is CONTRAINDICATED in this situation?

Добутамін Dobutamine

Каптоприл Captopril

Фуросемід Furosemide

Клофелін Clofelin

Рибоксин Riboxin

94 / 200
Хворий 27-ми років знаходиться на стаціонарному лікуванні з приводу вірусного гепатиту A 27-year-old patient is undergoing inpatient treatment for viral hepatitis

20 мл 40% глюкози 20 ml of 40% glucose

100 мл ізотонічного розчину 100 ml of isotonic solution

На 7-й день лікування після вечері великою кількістю м’ясної їжі у хворого пропав апетит, з’явились блювання, гикавка, запаморочення. Хворий збуджений. Шкіра жовтяничного кольору, геморагії на шкірі, тремор м’язів, сповільнена мова. Розміри печінки зменшилися. Діагностовано гостру печінкову недостатність. Введення якого препарату, що використовується в лікуванні печінкової недостатності, дозволить зменшити ендогенну інтоксикацію? On the 7th day of treatment, after dinner with a large amount of meat, the patient lost his appetite, vomiting, hiccups, and dizziness appeared. The patient is excited. The skin of the jaundice color, hemorrhages on the skin, muscle tremors, slowed speech. The size of the liver has decreased. Acute liver failure has been diagnosed. The introduction of which drug used in the treatment of liver failure will reduce endogenous intoxication?

Орнідазол 6- Небіволол Ornidazole 6- Nebivolol

Лактулоза Lactulose

95 / 200
У хворого 72-х років раптово розвинулось блювання кров’ю з крововтратою до 1,0 л. Об’єктивно: AT- 70/40 мм рт.ст., ЧСС-112/хв. ЦВТ- 0. Які інфузійні середовища першочергово треба застосувати для лікування? A 72-year-old patient suddenly developed vomiting of blood with blood loss up to 1.0 L. Objectively: AT- 70/40 mm Hg ., heart rate-112/min. CVT- 0. What infusion media should be used primarily for treatment?

Кристалоїдні розчини Crystalloid solutions

Розчини глюкози Glucose solutions

Жирові емульсії Fat emulsions

Колоїдні розчини Colloidal solutions

Свіжозаморожена плазма Fresh-frozen plasma

96 / 200
Жінку 77-ми років доставлено до лікарні непритомною. Неврологічний статус: лівобічна параплегія. Протромбін за Квіком становить 108%. На КТ-сканах відзначається: у правій півкулі головного мозку ділянка з нерівномірним зниженням оптичної щільності. Який патологічний стан розвинувся у пацієнтки? A 77-year-old woman was brought to the hospital unconscious. Neurological status: left-sided paraplegia. Quick's prothrombin is 108%. CT scans show: in the right hemisphere of the head an area of ​​the brain with an uneven decrease in optical density. What pathological condition has developed in the patient?

Ішемічний інсульт Ischemic stroke

Геморагічний інсульт Hemorrhagic stroke

Менінгоенцефаліт Meningoencephalitis

Субарахноїдальний крововилив Subarachnoid hemorrhage

Метастази у головний мозок Metastases in the brain

97 / 200
Пацієнт 59-ти років проходить курс реабілітаційного лікування після перенесеного інфаркту міокарда. Гемодинамічні показники стабільні. Фракція викиду 50%. Які засоби профілактики раптової серцевої смерті потрібно застосувати у даного пацієнта? A 59-year-old patient is undergoing rehabilitation treatment after a myocardial infarction. Hemodynamic parameters are stable. Ejection fraction is 50%. What means of prevention of sudden cardiac death should be used in this patient the patient?

Діуретики, іАПФ, статини Diuretics, ACE inhibitors, statins

Антагоністи кальцію, ІАПФ, статини Calcium antagonists, ACE inhibitors, statins

β-адреноблокатори, іАПФ, статини β-blockers, ACE inhibitors, statins

Аміодарон, α-адреноблокатори Amiodarone, α-blockers

Хірургічна корекція Surgical correction

98 / 200
На 4-ту добу після переливання несумісної по групі крові, у хворого різко зменшився діурез, розвинулася анурія, різко погіршився загальний стан, підвищився артеріальний тиск. При лабораторних дослідженнях: креатинін плазми - 680 мкмоль/л, сечовина плазми - 24 мкмоль/л. Яке захворювання і яку його стадію можна припустити в першу чергу? On the 4th day after the transfusion of incompatible blood group, the patient's diuresis decreased sharply, anuria developed, the general condition deteriorated sharply, blood pressure increased. During laboratory tests : plasma creatinine - 680 μmol/l, plasma urea - 24 μmol/l. What disease and what stage of it can be assumed in the first place?

Постгеморагічна гостра ниркова недостатність, анурія Posthemorrhagic acute renal failure, anuria

Гемотрансфузійний шок, постренальна гостра ниркова недостатність, анурія Hemotransfusion shock, postrenal acute renal failure, anuria

Анафілактичний шок, гостра ниркова недостатність, анурія Anaphylactic shock, acute renal failure, anuria

Гострий інтерстиціальний нефрит, обтураційна анурія Acute interstitial nephritis, obstructive anuria

Гемотрансфузійна гостра ниркова недостатність, анурія Hemotransfusion acute renal failure, anuria

99 / 200
Дитина 11-ти років грала у футбол та під час падіння отримала травму. Встановлено попередній діагноз: закритий вивих правого передпліччя. Який обсяг допомоги повинен надати лікар на місці пригоди? An 11-year-old child was playing football and was injured during a fall. The preliminary diagnosis was established: closed dislocation of the right forearm. What amount of help should the doctor provide at the scene of the accident ?

Косинкова пов’язка на праву верхню кінцівку Handkerchief bandage on the right upper limb

Восьмиподібна пов’язка на правий ліктьовий суглоб Eight-shaped bandage on the right elbow joint

Знеболювання, транспортна іммобілізація косинкою Anesthesia, transport immobilization with a scarf

Знеболювання, спроба вправлення вивиху Anesthesia, attempt to repair dislocation

Знеболювання, транспортна іммобілізація шиною Anesthesia, transport immobilization with a splint

100 / 200
Пішохід 62-х років під час ожеледиці послизнувся та впав на руку на бордюр тротуару. Є локалізований біль у ділянці верхньої третини лівого передпліччя. Спостерігаються тахікардія, підвищення артеріального тиску, припухлість, деформація, синець та крепітація в ділянці верхньої третини лівого передпліччя, рухи кінцівки неможливі. Можна припустити пошкодження верхньої третини лівого передпліччя. Як слід транспортувати хворого? A 62-year-old pedestrian slipped and fell on his hand on the curb of the sidewalk during the ice. There is localized pain in the upper third of the left forearm. Tachycardia, increased blood pressure are observed , swelling, deformation, bruising and crepitus in the area of ​​the upper third of the left forearm, movements of the limb are impossible. Damage to the upper third of the left forearm can be assumed. How should the patient be transported?

Наклавши шину Крамера з фіксацією плечового, ліктьового, променевозап’ясткового суглобів Having applied Kramer's splint with fixation of shoulder, elbow, radiocarpal joints

Наклавши пов’язку Дезо з валиком в пахвовій ділянці Having applied a Deso bandage with a roller in the armpit

Прибинтувавши праву верхню кінцівку до грудної клітки та здорової кінцівки Bandaging the right upper limb to the chest and healthy limb

В напівсидячому положенні з палкою в ліктьових згинах In a semi-sitting position with a stick in the elbows

Наклавши шину Крамера від верхньої третини плеча до п’ястно-фалангових суглобів Putting a Kramer splint from the upper third of the shoulder to the carpo-phalangeal joints

101 / 200
У хворого під час операції на шлунку було ушкоджено селезінкову артерію, в результаті чого виникла масивна кровотеча, яка призвела до зупинки кровообігу. Найдоцільніше проводити: The patient's splenic artery was damaged during a stomach operation, as a result of which massive bleeding occurred, which led to the cessation of blood circulation. It is most expedient to carry out:

Непрямий масаж серця Indirect heart massage

Абдомінальну компресію Abdominal compression

Введення гідрокортизону Injection of hydrocortisone

Прямий масаж серця Direct heart massage

Електрокардіостимуляцію Electronic cardiostimulation

102 / 200
У хворого після автомобільної аварії з’явилися скарги на різку задишку, кашель з кров’ю. Об’єктивно: шкіра бліда, ціанотична. Підшкірна емфізема в ділянці грудної клітки, живота, шиї справа. Аускультативно: справа дихання не вислуховується. Ps-130/хв., AT95 /60 мм рт.ст., Ht- 0,25, Hb- 88 г/л. Які першочергові заходи лікаря на до-госпітальному етапі? After a car accident, the patient complained of shortness of breath, coughing up blood. Objectively: the skin is pale, cyanotic. Subcutaneous emphysema in the chest area chest, abdomen, neck on the right side. Auscultation: breathing is not heard on the right side. Ps-130/min., AT95 /60 mm Hg, Ht- 0.25, Hb- 88 g/l. What are the primary measures of the doctor to hospital stage?

Трансфузія еритромаси відповідно до групової і резусної належності Erythromass transfusion according to group and rhesus affiliation

Активна аспірація повітря справа Active air aspiration on the right

Дофамін 2-5 мкг/кг/хв., інфузія колоїдних розчинів Dopamine 2-5 μg/kg/min, infusion of colloidal solutions

Колоїди, еритроцитарна маса, кристалоїди Colloids, erythrocyte mass, crystalloids

Негайна штучна вентиляція легень Immediate CPR

103 / 200
Хворий лікувався в гастроентерологічному відділенні з діагнозом: алкогольний цироз печінки. В останню добу став загальмованим, на запитання не відповідає. Об’єктивно: ’’печінковий запах” із рота, посилення жовтушності шкіри та склер. Відзначається зростання рівня трансаміназ, тимолової проби. Які заходи необхідно провести першочергово? The patient was treated in the gastroenterology department with a diagnosis of alcoholic cirrhosis of the liver. In the last day, he became sluggish, does not answer questions. Objectively: 'liver smell' with mouth, increased yellowness of the skin and sclera. There is an increase in the level of transaminases, thymol test. What measures should be taken as a priority?

Проведення дезінтоксикаційної терапії Carrying out detoxification therapy

Призначення антибіотиків Prescription of antibiotics

Призначення інсуліну Prescription of insulin

Призначення ферментних препаратів Prescription of enzyme preparations

Призначення жовчогінних засобів Prescription of choleretics

104 / 200
Хворий 56-ти років при падінні у відкритий каналізаційний люк дістав закритий вивих правого плеча. Який вид транспортної іммобілізації слід використати? A 56-year-old patient suffered a closed dislocation of the right shoulder when falling into an open sewer hatch. What type of transport immobilization should be used?

Драбинчата шина від променевозап’яс-ткового суглоба до кута здорової лопатки Ladder splint from the radiocarpal joint to the angle of the healthy scapula

Іммобілізація підручними засобами Immobilization by improvised means

Фіксація верхньої кінцівки до тулуба Fixation of the upper limb to the trunk

Косинкова іммобілізація Kosyn immobilization

M ’яка пов’язка Дезо і Вельпо M 'what a bandage Dezo and Velpo

105 / 200
На медичному пункті полку хворий 25-ти років зі скаргами на слабкість, нудоту, запаморочення. Близько 3 годин був ядерний вибух. Шкіра бліда, волога, ЧД- 28/хв., периферичні лімфовузли не збільшені, геморагій немає, дихання везикулярне, тони приглушені; Ps- 106/хв., AT-110/70 мм рт.ст. Живіт болісний по ходу тонкого кишківника. Набряків немає. Показник індивідуального дозиметра 3,9 Гр. Чим обумовлені клінічні прояви захворювання? At the regimental medical center, a 25-year-old patient complains of weakness, nausea, dizziness. There was a nuclear explosion about 3 hours ago. The skin is pale, moist, BH-28 /min., peripheral lymph nodes are not enlarged, no hemorrhages, breathing is vesicular, tones are muffled; Ps- 106/min., AT-110/70 mm Hg. Abdomen is painful along the course of the small intestine. There is no swelling. Individual dosimeter reading 3, 9 Gr. What causes the clinical manifestations of the disease?

Гостра променева хвороба IV ступеня Acute radiation sickness of the IV degree

Гостра токсикоінфекція Acute toxic infection

Гостра променева хвороба І ступеня Acute radiation sickness of the 1st degree

Гостра променева хвороба II ступеня Acute radiation sickness of the II degree

Гостра променева хвороба III ступеня Acute radiation sickness of the III degree

106 / 200
У пацієнтки 31-го року на 10-ту добу після пологів раптово з’явилися різкі болі в клубовій і пахвинній областях зліва, що поширюються на ліве стегно, відчуття розпирання, тягаря в лівій нижній кінцівці. Температура тіла - 39,6°С, супроводжується ознобом. Об’єктивно: ціаноз і набряк передньої черевної стінки, статевих органів і поперекової області. Які антибактеріальні препарати є препаратами вибору при лікуванні цього стану? On the 10th day after childbirth, a 31-year-old patient suddenly developed sharp pains in the iliac and inguinal regions on the left, spreading to the left thigh, feeling distention, heaviness in the left lower limb. Body temperature - 39.6°C, accompanied by chills. Objectively: cyanosis and swelling of the anterior abdominal wall, genitals and lumbar region. What antibacterial drugs are the drugs of choice in the treatment of this condition?

Амоксицилін + клавуланова кислота 1,2 г внутрішньовенно 3 рази на добу Amoxicillin + clavulanic acid 1.2 g intravenously 3 times a day

Гентаміцин 3-5 мг/кг на добу в 1-2 внутрішньом’язових введеннях Gentamicin 3-5 mg/kg per day in 1-2 intramuscular injections

Лінкоміцин 0,6 г внутрішньом’язово З рази на добу Lincomycin 0.6 g intramuscularly 3 times a day

Нетілміцин 5-7,5 мг/кг на добу в 1-2 внутрішньом’язових введеннях Netilmicin 5-7.5 mg/kg per day in 1-2 intramuscular injections

Кліндаміцин 0,6 г внутрішньом’язово З рази на добу Clindamycin 0.6 g intramuscularly 3 times a day

107 / 200
Хворого 28-ми років екстрено доставлено до операційної з приводу профузної шлунковокишкової кровотечі. Загальний стан тяжкий, свідомість - оглушення, АТ-80/40 мм рт.ст., ЧСС- 110/хв., олігурія. Вкажіть об’єм крововтрати: A 28-year-old patient was urgently taken to the operating room due to profuse gastrointestinal bleeding. The general condition is serious, consciousness - stupor, blood pressure 80/40 mm Hg. , heart rate - 110/min., oliguria. Specify the volume of blood loss:

Крововтрати немає No blood loss

20-25% дефіциту ОЦК (1250-1750 мл ) 20-25% deficit of BCC (1250-1750 ml)

30-40% дефіциту ОЦК (>1750 мл) 30-40% BCC deficiency (>1750 ml)

>40% дефіциту ОЦК (>2000 мл ) >40% BCC deficiency (>2000 ml)

15% дефіциту ОЦК (750-1250 мл ) 15% deficit of BCC (750-1250 ml)

108 / 200
При наданні невідкладної допомоги дитині із крововтратою до 10% ОЦК лікарю необхідно відновити обсяг циркулюючої плазми. Оберіть оптимальний препарат: When providing emergency care to a child with blood loss of up to 10% of BCC, the doctor needs to restore the volume of circulating plasma. Choose the optimal drug:

Розчин глюкози Glucose solution

Ізотонічний сольовий розчин Isotonic saline

Цільна кров Whole blood

Свіжозаморожена плазма Fresh-frozen plasma

Реополіглюкін Rheopoliglyukin

109 / 200
У дитини 12-ти років з нервово-артритичним діатезом після вживання в їжу значної кількості м’яса з кетчупом раптово виник гострий переймоподібний біль у животі та попереку, який іррадіює у статеві органи. При сечовипусканні отримано каламутну сечу з домішками крові. Найбільш імовірна причина гематурії? A 12-year-old child with neuro-arthritic diathesis, after eating a large amount of meat with ketchup, suddenly developed acute spasm-like pain in the abdomen and lower back, which radiates to the genitals. When urinating, cloudy urine with blood impurities was obtained. The most likely cause of hematuria?

Нефритичний синдром Nephritic syndrome

Гострий гломерулонефрит Acute glomerulonephritis

Ниркова колька Renal colic

Травматичне ушкодження сечовивідних шляхів Traumatic injury of the urinary tract

Геморагічний цистит Hemorrhagic cystitis

110 / 200
Солдат доставлений у непритомному стані до медичної роти з казарми. Зі слів товаришів по службі, приблизно за 6 годин до цього він прав обмундирування в закритому приміщенні в розчині технічної рідини. Об’єктивно: непритомний, однак реакція на больові подразники збережена. Шкіра і видимі слизові оболонки бліді, у повітрі, що видихається, солодкий запах. Ps- 80/хв., ритмічний, тони серця звучні, AT- 90/60 мм рт.ст., ЧД- 18/хв., дихання везикулярне, живіт м’який, неболючий. Який найбільш імовірний діагноз? The soldier was brought in an unconscious state to the medical company from the barracks. According to his comrades, about 6 hours before, he washed his uniform in a closed room in a solution of technical fluid . Objectively: unconscious, but the reaction to painful stimuli is preserved. The skin and visible mucous membranes are pale, there is a sweet smell in the exhaled air. Ps- 80/min., rhythmic, heart sounds are sonorous, AT- 90/60 mm Hg .st., BH- 18/min., vesicular breathing, soft, painless abdomen. What is the most likely diagnosis?

Отруєння етиленгліколем важкого ступеня Severe ethylene glycol poisoning

Пероральне отруєння хлорованими вуглеводнями важкого ступеня Severe oral chlorinated hydrocarbon poisoning

Отруєння ароматичними вуглеводнями середнього ступеня Moderate aromatic hydrocarbon poisoning

Інгаляційне отруєння тетраетилсвинцем важкого ступеня Severe tetraethyl lead inhalation poisoning

Інгаляційне отруєнням хлорованими вуглеводнями Inhalation poisoning with chlorinated hydrocarbons

111 / 200
Сержант отримав вогнепальне поранення грудей. Встановлено діагноз: кульове сліпе проникаюче поранення правої половини грудей з пошкодженням легені. Правобічний закритий гемопневмоторакс. ІЬ-стра крововтрата. Гостра дихальна недостатність II ступеня. Виконане дренування плевральної порожнини за Бюлау. Назвіть критерії внутрішньоплевральної кровотечі, що продовжується: The sergeant received a gunshot wound to the chest. The diagnosis was established: a bullet blind penetrating wound of the right half of the chest with damage to the lung. Right-sided closed hemopneumothorax. IB-grade blood loss. Acute respiratory failure II degree. Performed drainage of the pleural cavity according to Bülau. Name the criteria for intrapleural bleeding that continues:

Негативна проба Квеккенштедта Negative Queckenstedt test

Позитивна проба Стуккея Positive Stuckey test

Негативна проба Рувілуа Negative Rouvilois test

Позитивна проба Квеккенштедта Positive Queckenstedt test

Позитивна проба Рувілуа-Грегуара Positive Rouvilois-Gregoire test

112 / 200
Хворий 29-ти років захворів гостро 18 годин тому: після вживання заливної риби з’явилися нудота, повторне блювання, часті пінисті рідкі зеленуваті випорожнення, підвищення температури до 39°С. Об’єктивно: стан важкий. Блідий, млявий, тургор шкіри різко знижений, сухість слизової оболонки рота, AT- 80/40 мм рт.ст., пульс слабкого наповнення, 112/хв., живіт болючий в епігастрії, біляпупковій та ілеоцекальній ділянках, зменшення виділення сечі. Які невідкладні терапевтичні заходи? A 29-year-old patient became acutely ill 18 hours ago: nausea, repeated vomiting, frequent frothy liquid greenish stools, temperature rise to 39 ° C. Objectively: the condition is severe. Pale, lethargic, skin turgor is sharply reduced, dryness of the oral mucosa, AT- 80/40 mm Hg, weakly filling pulse, 112/min, abdominal pain in the epigastrium, near the umbilicus and ileocecal areas, decreased urine output. What are the urgent therapeutic measures?

Введення глюкокортикостероїдів Introduction of glucocorticosteroids

Парентеральне введення антибіотиків Parenteral administration of antibiotics

Внутрішньовенне введення розчину глюкози Intravenous administration of glucose solution

Внутрішньовенне введення сольових розчинів Intravenous administration of saline solutions

Внутрішньовенне введення реополіглю-кіну Intravenous administration of rheopolyglucin

113 / 200
У хворого 28-ми років субфебрильна температура, біль у животі, рідкі випорожнення з домішками слизу до 3-4 разів на добу. Хворіє протягом 3-х діб. Язик вологий, обкладений білим нальотом. Живіт м’який, болючий в лівій здухвинній ділянці, сигмоподібна кишка спазмована, болюча. Який метод дослідження буде найбільш інформативним для постановки діагнозу? A 28-year-old patient has low-grade fever, abdominal pain, loose stools with mucus up to 3-4 times a day. He has been ill for 3 days. The tongue is moist, coated with a white coating. The abdomen is soft, painful in the left pubic region, the sigmoid colon is spasmodic, painful. What research method will be the most informative for making a diagnosis?

Копрокультура Coproculture

Загальний аналіз крові General blood test

Копроцитограма Coprocytogram

Загальний аналіз сечі General urinalysis

РИГА RIGA

114 / 200
Хлопчик у віці 6 місяців хворіє на тяжку форму кашлюка: число нападів кашлю становить 25, напади тяжкі, тривалістю до 10 хвилин, мають 6-8 репризів, закінчуються блюванням. Один з нападів кашлю призвів до апное. Які невідкладні дії? A 6-month-old boy has a severe form of whooping cough: the number of coughing attacks is 25, the attacks are severe, last up to 10 minutes, have 6-8 repetitions, end with vomiting . One of the coughing fits led to apnea. What are the immediate actions?

Преднізолон в/м Prednisolone IV

Лобелін в/м Lobelin v/m

Налагодити подачу кисню Set up oxygen supply

Седуксен в/м Seduksen v/m

Штучне дихання (в ручному режимі) Artificial respiration (in manual mode)

115 / 200
У хворого 27-ми років впродовж року відмічаються постійні міоклонічні гіперкінези правої кінцівки, гіпертонус та гіпотрофічні порушення м’язів правої руки та шиї, періодичні загальні генералізовані судомні напади та фокальні сенсомоторні, тахікардія, пітливість, загальна слабкість, підвищена сонливість, коливання тиску. В анамнезі зловживання алкоголем, часті легкі ЧМТ, робота в будівельному загоні у Сибіру минулим літом. Який найбільш імовірний діагноз? A 27-year-old patient has had persistent myoclonic hyperkinesis of the right limb, hypertonus and hypotrophic disorders of the muscles of the right arm and neck, periodic general generalized convulsive seizures and focal sensorimotor, tachycardia, sweating, general weakness, increased drowsiness, blood pressure fluctuations. History of alcohol abuse, frequent mild TBI, work in a construction yard in Siberia last summer. What is the most likely diagnosis?

Алкогольна епілепсія Alcoholic epilepsy

Кліщовий енцефаліт із проявами кожев-ніковської епілепсії Tick-borne encephalitis with manifestations of Kozhev-Nikov epilepsy

Післятравматична епілепсія Post-traumatic epilepsy

Судомний синдром при пухлині мозку Seizure syndrome in brain tumor

Епілептична хвороба Epileptic disease

116 / 200
До інфекційного відділення надійшов хворий 20-ти років з ознаками дегідратації III ступеня. Спостерігаються судоми, блювання фонтаном, різке зниження артеріального тиску, тахікардія, температура тіла - 35,5°С. З якою швидкістю треба проводити в/в інфузію для відновлення водно-електролітного балансу? A 20-year-old patient with signs of III degree dehydration was admitted to the infectious department. Convulsions, fountain vomiting, a sharp drop in blood pressure, tachycardia, body temperature - 35, 5°C. At what speed should IV infusion be carried out to restore the water-electrolyte balance?

50-80 мл/хв. 50-80 ml/min.

30-50 мл/хв. 30-50 ml/min.

Швидкість не має значення Speed ​​doesn't matter

80-100 мл/хв. 80-100 ml/min.

100-120 мл/хв. 100-120 ml/min.

117 / 200
У хворого з тяжким перебігом пневмонії за клініко-епідеміологічними даними можна припустити легеневу форму сибірки. Який з методів профілактики захворювання серед представників медичного персоналу, які контактували з цим хворим, слід вважати найбільш ефективним? In a patient with a severe course of pneumonia, according to clinical and epidemiological data, it is possible to assume the pulmonary form of anthrax. should be considered the most effective?

Дератизація Deratization

Дезінсекція Disinsection

Вакцинація Vaccination

Методи поточної дезінфекції Current disinfection methods

Екстрена антибіотикопрофілактика Emergency antibiotic prophylaxis

118 / 200
Дівчинка 2-х місяців з ознаками гіпотрофії знаходилася на грудному вигодовуванні матір’ю, що страждала на полінаркоманію. Після припинення вживання опіатів терміном на 3 місяці вона знову ввела собі звичну дозу внутрішньовенно. Після годування груддю дитина стала кволою, зблідла, дихання 10-12/хв. За медичною допомогою мати не зверталась. Через 40 хвилин у дитини виникло припинення крово- обігу, яке призвело до біологічної смерті. Який тип дихальної недостатності спричинив смерть немовляти? A 2-month-old girl with signs of hypotrophy was breastfed by her mother, who suffered from polydrug addiction. After stopping the use of opiates for 3 months, she again injected herself the usual dose intravenously. After breastfeeding, the child became weak, pale, breathing 10-12/min. The mother did not seek medical help. After 40 minutes, the child suffered a circulatory arrest, which led to biological death. What type of respiratory failure caused death babies?

Центральний Central

Нервово-м’язовий Neuromuscular

Рестриктивний Restrictive

Торакоабдомінальний Thoracoabdominal

Констріктивний Constrictive

119 / 200
Чоловік 64-х років знаходиться 2 доби на стаціонарному лікуванні з приводу гострого інфаркту міокарда. Впродовж останніх 8 годин з’явилась гостра затримка сечі. Із анамнезу відомо, що хворий страждає на доброякісну гіперплазію передміхурової залози. Залишковий об’єм сечі в сечовому міхурі 600 мл. Спроба проведення уретрального катетера в сечовий міхур невдала. Вкажіть найбільш раціональну тактику надання допомоги хворому: A 64-year-old man has been hospitalized for 2 days due to an acute myocardial infarction. During the last 8 hours, acute urinary retention has appeared. From the anamnesis, it is known that the patient suffers from benign hyperplasia of the prostate gland. The residual volume of urine in the bladder is 600 ml. An attempt to insert a urethral catheter into the bladder was unsuccessful. Specify the most rational tactics for providing assistance to the patient:

Відкрита простатектомія Open prostatectomy

Консервативна протизапальна терапія Conservative anti-inflammatory therapy

Дренування сечового міхура через надлобкову норицю Bladder drainage through suprapubic fistula

Трансуретральна резекція передміхурової залози Transurethral resection of the prostate

Консервативна гормональна терапія Conservative hormone therapy

120 / 200
Громадянин іноземної держави доставлений в приймальне відділення інфекційної лікарні у вкрай важкому стані. Епідеміологічний анамнез: працює в вірусологічній лабораторії Об’єктивно: температура тіла - 40,5°С. Свідомість спутана. AT- 100/60 мм рт.ст., ЧСС120 /хв. На шкірі обличчя, тулуба, верхніх та нижніх кінцівок множинні багатокамерні пустули з пупкови-дним вдавленням по центру пухирця. Шкіра обличчя та кінцівок різко набрякла. Гнійний кон’юнктивіт. При аускультації: в легенях жорстке дихання. Тони серця приглушені. Живіт м’який, болісний під час пальпації в нижніх відділах. Печінка +3 см. Пальпується край селезінки. Випорожнення 2 рази на добу з домішками крові. Сеча мутна, з червонуватим відтінком. На який термін встановлюється обов’язкове медичне спостереження за контактними особами? A citizen of a foreign country was brought to the reception department of an infectious disease hospital in an extremely serious condition. Epidemiological history: works in a virological laboratory Objectively: body temperature - 40.5°С . Consciousness is confused. AT- 100/60 mm Hg, heart rate 120/min. On the skin of the face, trunk, upper and lower limbs, multiple multichamber pustules with an umbilical-fundal depression in the center of the blister. The skin of the face and limbs is sharply swollen. Purulent con conjunctivitis. On auscultation: hard breathing in the lungs. Heart sounds are muffled. The abdomen is soft, painful during palpation in the lower parts. Liver +3 cm. The edge of the spleen is palpated. Defecation 2 times a day with blood impurities. Urine cloudy, with with a reddish tint. For what period is mandatory medical observation of contact persons established?

На 5 днів For 5 days

На 10 днів For 10 days

На 21 день For 21 days

На 14 днів For 14 days

Медичне спостереження не встановлюється Medical monitoring is not set

121 / 200
Вагітна 40-ка років в терміні 37 тижнів скаржиться на різкий головний біль, зниження зору, біль в епігастрїї, нудоту. AT на обох руках 190/120 мм рт.ст. В сечі білок 5 г/л. Раптово у вагітної з’явився акроціаноз, розлад дихання. Які реанімаційні заходи необхідно провести вагітній негайно? A 40-year-old pregnant woman at 37 weeks complains of a sharp headache, decreased vision, pain in the epigastrium, nausea. AT on both arms 190/120 mm Hg .st. Protein in the urine is 5 g/l. Suddenly, a pregnant woman developed acrocyanosis, breathing disorder. What resuscitation measures should be carried out for a pregnant woman immediately?

Призначити протисудомні препарати Prescribe anticonvulsants

Штучна вентиляція легень Artificial lung ventilation

Гіпотензивна терапія Hypotensive therapy

Призначити сечогінні препарати Prescribe diuretics

Седативна терапія Sedation therapy

122 / 200
Хворий 42-х років захворів гостро: підвищення температури тіла до 39°С, головний та м’язовий біль. На 5-й день захворювання: обличчя гіперемоване, 'кролячі очі', на бічних поверхнях грудної клітки рясний поліморфний розеольозно-петехіальний висип. Тони серця глухі, Ps- 120/хв., АТ-110/70 мм рт.ст. Язик сухий, обкладений білим нальотом з відбитками зубів. Іепато-спленомегалія. Який найбільш імовірній діагноз? A 42-year-old patient became acutely ill: an increase in body temperature to 39°C, headache and muscle pain. On the 5th day of the illness: the face is hyperemic, 'rabbit eyes', on the lateral surfaces of the chest there is an abundant polymorphic roseolous-petechial rash. Heart sounds are dull, Ps - 120/min., BP - 110/70 mm Hg. The tongue is dry, covered with a white plaque with imprints of teeth. Hepato- splenomegaly. What is the most likely diagnosis?

Лептоспіроз Leptospirosis

Грип Flu

Черевний тиф Typhoid

Менінгококцемія Meningococcemia

Висипний тиф Typhoid

123 / 200
Хвора на варикозну хворобу нижніх кінцівок впала. Виникла сильна кровотеча із варикозно розширеного вузла в середній третині лівої гомілки. Як зупинити кровотечу на догоспітальному етапі? A patient with varicose veins of the lower extremities fell. There was severe bleeding from a varicose node in the middle third of the left lower leg. How to stop the bleeding at the pre-hospital stage?

Накласти джгут на місце ушкодження Apply a tourniquet to the injury

Накласти джгут вище місця ушкодження Apply a tourniquet above the injury

Накласти стискаючу пов’язку на місце ушкодження Put a compression bandage on the injury

Накласти джгут нижче місця ушкодження Put a tourniquet below the injury

Накласти стискаючу пов’язку вище місця ушкодження Put a compression bandage above the injury

124 / 200
У хворого 26-ти років після ДТП виявлено відкриті переломи обох гомілок в середній третині та закриту травму грудної клітки з пошкодженням легені й напруженим пневмотораксом. Яка першочергова тактика? A 26-year-old patient was found to have open fractures of both lower legs in the middle third and a closed chest injury with lung damage and a tense pneumothorax after a road accident. What is the primary tactic?

Консервативне лікування, спостереження Conservative treatment, observation

Екстрена торакотомія Emergency thoracotomy

Інтубація трахеї Tracheal intubation

Проведення штучної вентиляції легень Carrying out artificial lung ventilation

Дренування плевральної порожнини Drainage of the pleural cavity

125 / 200
У пологовий будинок доставлена першовагітна 20-ти років з Rh(-) фактором у терміні вагітності 14 тижнів зі скаргами на ниючий біль у низу живота, значні кров’янисті виділення з статевих шляхів із згустками, що з’явились після ДТП. При дообстеженні встановлено повний аборт. З метою профілактики Rh-імунізації в якій дозі слід ввести aHTH-Rh(D) імуноглобулін і за який час? A 20-year-old primiparous woman with the Rh(-) factor was delivered to the maternity hospital at 14 weeks of pregnancy with complaints of aching pain in the lower abdomen, significant bleeding discharge from the genital tract with clots that appeared after a road accident. A complete abortion was established during the additional examination. In order to prevent Rh-immunization, in what dose should aHTH-Rh(D) immunoglobulin be administered and for what time?

300 мкг, до 70 годин 300 mcg, up to 70 hours

75 мкг, до 70 годин 75 mcg, up to 70 hours

150 мкг, до 48 годин 150 mcg, up to 48 hours

120 мкг, до 70 годин 120 mcg, up to 70 hours

300 мкг, до 48 годин 300 mcg, up to 48 hours

126 / 200
Народилась недоношена дитина, геста-ційний вік 32 тижні, з вагою 1700 г, рівень глюкози крові становить 2,1 ммоль/л. У дитини відмічається порушення стану, судомна готовність. Виберіть першочергову допомогу для дитини: A premature baby was born, gestational age 32 weeks, with a weight of 1700 g, the blood glucose level is 2.1 mmol/l. The child has a condition disorder, convulsive readiness Choose the first aid for the child:

Фенобарбітал, 20 мг/кг Phenobarbital, 20 mg/kg

10% розчин глюкози, 2 мл/кг 10% glucose solution, 2 ml/kg

5% розчин глюкози, 4 мл/кг 5% glucose solution, 4 ml/kg

25% розчин MqSOa, Імл/кг 25% MqSOa solution, Iml/kg

0,9% розчин натрію хлориду, 10 мл/кг 0.9% sodium chloride solution, 10 ml/kg

127 / 200
До хірургічного відділення поступив потерпілий 37-ми років з великою розчавленою раною лівого стегна, через 4 години після травми. Що є першочерговою умовою успішної профілактики газової гангрени у потерпілого? A 37-year-old victim was admitted to the surgical department with a large crushed wound of the left thigh, 4 hours after the injury. What is the primary condition for the successful prevention of gas gangrene in the victim?

Промивання рани розчином перекису водню 6% Wound washing with 6% hydrogen peroxide solution

Видалення нежиттєздатних тканин та своєчасна хірургічна обробка рани Removal of nonviable tissues and timely surgical treatment of the wound

Введення специфічної сироватки 30 000 ОД Introduction of specific serum 30,000 units

Введення специфічної сироватки 3 000 ОД Introduction of specific serum 3,000 units

Інфільтрація м’яких тканин довкола рани розчином антибіотиків Infiltration of soft tissues around the wound with antibiotic solution

128 / 200
Породілля 23-х років, на другий день після пологів відчула наростаючий біль в правій нозі, важкість. Об’єктивно: стан хворої середньої важкості. Ps- 100/хв., температура 37,З°С, тони серця приглушені, ритм синусовий, в легенях дихання везикулярне, 18-20/хв. Ліва нога на всьому протязі набрякла, синюшна, порівняно з правою температура знижена. Пульсація артерій лівої ноги на всьому протязі пальпується, ослаблена. Який найбільш імовірний діагноз? A 23-year-old woman in labor, on the second day after giving birth, felt increasing pain in her right leg, heaviness. Objectively: the condition of the patient is of medium severity. Ps- 100/ min., temperature 37.3°С, muffled heart sounds, sinus rhythm, vesicular breathing in the lungs, 18-20/min. The left leg is swollen throughout, bluish, compared to the right, the temperature is lower. Pulsation of the arteries of the left leg throughout palpable, weakened. What is the most likely diagnosis?

Гострий тромбофлебіт вен правої нижньої кінцівки Acute thrombophlebitis of the veins of the right lower limb

Гострий клубово-стегновий флеботромбоз Acute iliac-femoral phlebothrombosis

Тромбоемболія клубової артерії Thromboembolism of the iliac artery

Тромбоемболія стегнової артерії Thromboembolism of the femoral artery

Тромбофлебіт глибоких вен правої нижньої кінцівки, післяпологовий сепсис Thrombophlebitis of the deep veins of the right lower limb, postpartum sepsis

129 / 200
При проведенні закритого масажу серця людини 76-ти років, що знаходиться у стані клінічної смерті, необхідно здійснювати компресії з частотою: When performing a closed heart massage of a 76-year-old person who is in a state of clinical death, it is necessary to perform compressions with the frequency:

Не менше 80 на 1 хвилину At least 80 for 1 minute

Не менше 40 на 1 хвилину At least 40 for 1 minute

Не менше 60 на 1 хвилину At least 60 for 1 minute

Не менше 120 на 1 хвилину At least 120 for 1 minute

Не менше 100 на 1 хвилину At least 100 for 1 minute

130 / 200
Хворий 42-х років, співробітник гаражу, доставлений у лікарню у стані коми III ступеня. Об’єктивно: клонічні судоми, безконтрольне сечовиділення, дихання Чейн-Стокса, тахікардія, гіперемія обличчя та слизових оболонок. Який вид отруєння має місце? A 42-year-old patient, a garage worker, was brought to the hospital in a third-degree coma. Objectively: clonic convulsions, uncontrolled urination, Cheyne-Stokes breathing, tachycardia, hyperemia of the face and mucous membranes. What kind of poisoning is taking place?

Тетраетилсвинцем Tetraethyl lead

Аміаком Ammonia

Випарами бензину Gasoline fumes

Чадним газом With carbon monoxide

Парами ртуті Mercury vapors

131 / 200
Хворий у важкому стані. У хворого розвинувся пронос (рідкі випорожнення до 20 разів на добу кількістю 5-6 літрів), загальна слабкість, запаморочення. Останню добу приєдналось блювання 'фонтаном', сечовипускання припинилось. Об’єктивно: пацієнт загальмований, на питання відповідає мляво. Риси обличчя загострені. Шкіра бліда, має мармуровий малюнок, тургор знижений. Язик сухий. Температура тіла -35,9°С, AT- 80/40 мм рт.ст. Серцеві тони ритмічні, приглушені, ЧСС- 110/хв., судоми м’язів кінцівок. Живіт м’який. Важкий стан пацієнта обумовлює: The patient is in serious condition. The patient developed diarrhea (liquid stools up to 20 times a day in the amount of 5-6 liters), general weakness, dizziness. Vomiting has joined the last day by a 'fountain', urination has stopped. Objectively: the patient is inhibited, he answers questions sluggishly. Facial features are sharpened. The skin is pale, has a marble pattern, turgor is reduced. The tongue is dry. Body temperature -35.9°С, AT- 80/40 mm Hg. Heart sounds are rhythmic, muffled, heart rate - 110/min., spasms of the muscles of the limbs. Abdomen is soft. The patient's serious condition is caused by:

Тромбоз мезентеріальних судин Thrombosis of mesenteric vessels

Гіповолемічний шок Hypovolemic shock

Гостра затримка сечі Acute urinary retention

Гостре запалення очеревини Acute peritonitis

Кардіогенний шок Cardiogenic shock

132 / 200
Хворий 74-х років лікувався в хірургічному стаціонарі з приводу тромбофлебіту судин нижніх кінцівок. Одержував антикоагулянтну терапію. Одразу після лікування потрапив в ДТП. У хворого виникла масивна артеріальна кровотеча в ділянці правого стегна. Бригада швидкої медичної допомоги, яка прибула на місце події, діагностувала геморагічний шок. Раптово хворий втратив свідомість. На кардіомоніторі - асистолія. Які особливості проведення реанімаційних заходів в даному випадку? A 74-year-old patient was treated in a surgical hospital for thrombophlebitis of the vessels of the lower extremities. He received anticoagulant therapy. Immediately after the treatment, he got into a car accident. The patient had massive arterial bleeding in the area of ​​the right thigh. The emergency medical team that arrived at the scene diagnosed hemorrhagic shock. The patient suddenly lost consciousness. Asystole was shown on the cardiomonitor. What are the features of resuscitation measures in this case?

Накласти джгут вище ділянки кровотечі та розпочати комплекс серцево-легеневої реанімації Apply a tourniquet above the bleeding area and begin cardiopulmonary resuscitation

Нанести прекардіальний удар Deliver precardiac shock

Розпочати комплекс серцево-легеневої реанімації Start cardiopulmonary resuscitation complex

Накласти асептичну пов’язку та розпочати комплекс серцево-легеневої реанімації Apply an aseptic bandage and start cardiopulmonary resuscitation

Виконати електричну дефібриляцію Perform electrical defibrillation

133 / 200
Жінка 27-ми років, що страждає на цукровий діабет впродовж 9-ти років, доставлена в відділення інтенсивної терапії на 24 тижні вагітності непритомною. Із рота - різкий запах ацетону, дихання шумне, шкіра суха, тонус очних яблук і м’язів знижений. Ps- 142/хв., AT60 /25 мм рт.ст. Який найбільш імовірний діагноз? A 27-year-old woman, suffering from diabetes for 9 years, was brought to the intensive care unit at 24 weeks of pregnancy, unconscious. From the mouth - a sharp smell of acetone, breathing is noisy, the skin is dry, the tone of the eyeballs and muscles is reduced. Ps- 142/min., AT60 /25 mm Hg. What is the most likely diagnosis?

Печінкова кома Hepatic coma

Гіпоглікемічна кома Hypoglycemic coma

Кетонемічна кома Ketonemic coma

Лактатацидемічна кома Lactatacidemic coma

Гіперосмолярна кома Hyperosmolar coma

134 / 200
Солдат був в осередку ураження отруйною речовиною. Уражений евакуйований, введено 5 мл ОД% розчину атропіну протягом короткого часу Скаржиться на: загальну слабкість, спрагу, сухість у роті, охриплість голосу. Збуджений, поведінка агресивна. Шкіра обличчя гіперемована, зіниці розширені, D=S=6 мм. В легенях жорстке дихання, хрипів немає. Ps120 /хв., АТ-140/90 мм рт.ст., перистальтика ослаблена. Що могло б спричинити описану картину? The soldier was in the center of the lesion with a poisonous substance. The victim was evacuated, 5 ml of OD% solution of atropine was administered for a short time. He complains of: general weakness, thirst, dry mouth, hoarseness of the voice. Excited, aggressive behavior. The skin of the face is hyperemic, the pupils are dilated, D=S=6 mm. Hard breathing in the lungs, no wheezing. Ps120/min., BP-140/90 mm Hg, peristalsis is weakened. What could cause the described picture?

Атропінізація організму Atropinization of the body

Ураження дифосгеном Diphosgene exposure

Передозування реактиваторів холінесте-рази Overdose of cholinesterase reactivators

Ураження зоманом Infected by soman

Ураження зарином Sarin exposure

135 / 200
Вагітна 19-ти років в терміні ЗО тижнів поскаржилася на головний біль, порушення зору, блювання з домішками крові. Стан важкий, AT- 160/100 мм рт.ст., склери жовтушні, підшкірні крововиливи, набряки нижніх кінцівок, сеча геморагічного кольору. В крові: тромбоцитопенія. Який найбільш імовірний діагноз? A 19-year-old pregnant woman in 3 weeks complained of a headache, visual impairment, vomiting with blood impurities. The condition is severe, AT- 160/100 mm Hg. Art., sclerae are yellow, subcutaneous hemorrhages, swelling of the lower extremities, hemorrhagic urine. In the blood: thrombocytopenia. What is the most likely diagnosis?

HELP синдром HELP syndrome

ДВЗ-синдром DVZ-syndrome

Еклампсія Eclampsia

Гепатоз вагітних Hepatosis of pregnant women

Хвороба Боткіна Botkin's disease

136 / 200
Лікар швидкої допомоги, приїхавши на місце ДТП, діагностував закритий перелом правого стегна в середній третині. Яким повинен бути обсяг допомоги? The ambulance doctor, having arrived at the scene of the accident, diagnosed a closed fracture of the right hip in the middle third. What should be the amount of assistance?

Іммобілізація трьома шинами Крамера до кульшового суглоба Immobilization with three Kramer splints to the hip joint

Знеболення, іммобілізація шиною Дітеріхса, інфузійна терапія Anesthesia, immobilization with Dieterichs splint, infusion therapy

Знеболення наркотичними препаратами, фіксація пошкодженої кінцівки до здорової Anesthesia with narcotic drugs, fixation of a damaged limb to a healthy one

Знеболення введенням в місце перелому 1% р-ну новокаїну, іммобілізація шиною Дітеріхса, внутрішньовенне введення розчинів Anesthesia by injecting 1% novocaine solution into the fracture site, immobilization with Dieterichs bandage, intravenous injection of solutions

Іммобілізація трьома шинами Крамера до пахвинної ділянки Immobilization with three Kramer splints to the groin

137 / 200
Роділля, 40 тижнів, в кінці 1-го періоду пологів переведена на стіл для пологів. Після того, як вона лягла в горизонтальне положення на спину, обличчя її зблідло, вона вкрилася холодним потом і втратила свідомість. AT- 60/40 мм рт.ст., Ps- 132/хв. Яке ускладнення виникло? A woman in labor, 40 weeks, is transferred to the delivery table at the end of the 1st stage of labor. After she was placed in a horizontal position on her back, her face turned pale , she covered herself with a cold sweat and lost consciousness. AT- 60/40 mm Hg, Ps- 132/min. What complication occurred?

Синдром стиснення нижньої порожнистої вени Inferior vena cava compression syndrome

Тромбоемболія легеневої артерії Thromboembolism of the pulmonary artery

Внутрішня кровотеча Internal bleeding

Розрив матки Rupture of uterus

Відшарування плаценти Detachment of the placenta

138 / 200
Жінка 28-ми років звернулась в інфекційний стаціонар на 3-й день захворювання зі скаргами на підвищення температури до 39°С, болі в горлі, набряк у ділянці шиї. Об’єктивно: загальний стан середньої тяжкості. Шкірні покриви бліді, чисті. Слизова ротоглотки гіперемійована з ціанотичним відтінком. Мигдалики збільшені до II ступеня, вкриті сіруватим нальотом, який важко знімається, переходить на м’яке піднебіння, язичок. Пальпуються підщелепні лімфовузли, набряк м’яких тканин до середини шиї. Для якого захворювання характерна така клінічна картина? A 28-year-old woman applied to the infectious disease hospital on the 3rd day of the disease with complaints of a temperature rise to 39°C, sore throat, swelling in the neck area . Objectively: general condition of moderate severity. The skin is pale, clean. The mucous membrane of the oropharynx is hyperemic with a cyanotic shade. The tonsils are enlarged to the II degree, covered with a grayish coating that is difficult to remove, moving to the soft palate, uvula. The submandibular lymph nodes are palpated, swelling of soft tissues to the middle of the neck. What disease is characterized by such a clinical picture?

Дифтерія мигдаликів, поширена, плівчаста форма Diphtheria of the tonsils, common, membranous form

Інфекційний мононуклеоз Infectious mononucleosis

Ангіна Сімановського-Венсана Simanovsky-Vincent angina

Гострий лакунарний тонзиліт Acute lacunar tonsillitis

Епідемічний паротит Epidemic mumps

139 / 200
Під час оборонного бою в медичну частину полку стали евакуювати поранених різних форм важкості та характеру уражень. Чим повинні користуватись медичні працівники для позначення характеру ізоляції, місця надання допомоги, етапності та черги евакуації потерпілих? During the defensive battle, the wounded of various degrees of severity and nature of injuries were evacuated to the medical unit of the regiment. What should medical workers use to indicate the nature of isolation, the place of assistance, the stages and evacuation queues of victims?

Постраждалим надається повна допомога в порядку їх надходження Victims are provided with full assistance in the order of receipt

Сортування не проводиться No sorting

Усна домовленість Verbal Agreement

Розміщення постраждалих біля місць надання допомоги Placement of victims near places of assistance

Сортувальні марки Sorting stamps

140 / 200
Розвідкою було докладено про імовірну загрозу застосування супротивником радіаційної зброї. Який засіб з аптечки індивідуальної необхідно використати для профілактики променевого ураження особового складу військових? Intelligence was informed about the possible threat of the enemy using radiation weapons. Which medicine from the individual's first aid kit should be used to prevent radiation damage to military personnel?

Сульфодіметоксін Sulfodimethoxine

Тетрацикліну гідрохлорид Tetracycline hydrochloride

Афін Athens

Етаперазин Etaperazin

Цистамін Cystamine

141 / 200
Хлопець 20-ти років під час купання в річці раптово втратив свідомість та зник з поверхні води. Через 2 хвилини був доставлений до берега своїми однолітками. Об’єктивно: непритомний, шкіра вираженого фіолетово-синього кольору, велика кількість пінистих виділень з рота та з носа, дихання відсутнє, пульс на сонних артеріях не визначається. З чого потрібно починати комплекс СЛР? A guy in his 20s while swimming in the river suddenly lost consciousness and disappeared from the surface of the water. After 2 minutes, he was brought to the shore by his peers. Objectively: unconscious, the skin has a pronounced purple-blue color, a large amount of foamy secretions from the mouth and nose, there is no breathing, the pulse on the carotid arteries is not determined. Where should the CPR complex be started?

Очистити верхні дихальні шляхи Clear upper respiratory tract

Не проводити СЛР Do not perform CPR

Непрямий масаж серця Indirect heart massage

Прекардіальний удар Precardiac attack

ШВЛ VENTILATOR

142 / 200
У новонародженої дитини, яка народилась в асфіксії помірного ступеня, після тактильної стимуляції шкіри вздовж хребта спонтанне дихання не з’явилось. Які подальші дії у пологовому залі треба виконати? Spontaneous breathing did not appear after tactile stimulation of the skin along the spine in a newborn child who was born with moderate degree of asphyxia. What further actions should be performed in the delivery room?'

Почати штучну вентиляцію легень за допомогою мішка і маски Start artificial lung ventilation with bag and mask

Похлопати по п’ятах Clap your heels

Повторити погладжування вздовж хребта Repeat stroking along spine

Поплескування по підошві Pat on the sole

Зрошування холодною водою Irrigation with cold water

143 / 200
Після аварії на хімічному підприємстві рятувальники знайшли постраждалого, який скаржиться на головний біль, відчуття стиснення в грудній клітці, нудоту, часте блювання. На одежі й шкірі виявлено невідому хімічну речовину. Який об’єм медичної допомоги потрібно негайно надати постражд алому? After an accident at a chemical plant, rescuers found a victim complaining of a headache, chest tightness, nausea, frequent vomiting. An unknown chemical was found on his clothes and skin substance. What amount of medical aid should be provided immediately to the victim?

Накласти стерильні пов’язки на уражені ділянки шкіри, увести антидот Put sterile bandages on the affected areas of the skin, inject the antidote

Провести часткову санітарну обробку, увести антидот, підготовити постраждалого до евакуації Carry out partial sanitation, administer antidote, prepare victim for evacuation

Зняти забруднений одяг, провести повну санітарну обробку, накласти стерильні пов’язки Remove contaminated clothing, carry out full sanitation, apply sterile bandages

Надіти протигаз, винести постраждалого із осередку ураження, провести часткову санітарну обробку уражених ділянок шкіри Put on a gas mask, remove the victim from the affected area, carry out partial sanitation of the affected skin areas

Надіти протигаз, увести антидот, підготовити постраждалого до евакуації Put on a gas mask, administer an antidote, prepare the victim for evacuation

144 / 200
Роділля у першому періоді своєчасних пологів під час проведення внутрішнього акушерського дослідження поскаржилася на слабкість, шум в вухах, запаморочення, і раптово втратила свідомість. В першу чергу потрібно: A woman in labor in the first period of timely delivery during an internal obstetric examination complained of weakness, tinnitus, dizziness, and suddenly lost consciousness. First of all, you need:

Підняти нижні кінцівки Raise lower limbs

Забезпечити доступ свіжого повітря Ensure access to fresh air

Повернути жінку на лівий бік Turn the woman to the left side

Піднести розчин аміаку на ватному тампоні Put ammonia solution on a cotton swab

Посадити Plant

145 / 200
У вагітної під час стрімких пологів з’явились почуття задухи, ціаноз, падіння артеріального тиску. Через деякий час кров, що надходила з піхви, перестала згортатися. Який найбільш імовірний діагноз? During rapid childbirth, a pregnant woman felt suffocation, cyanosis, a drop in blood pressure. After some time, the blood coming from the vagina stopped clotting. What is the most probable diagnosis?

Емболія навколоплідними водами Amniotic fluid embolism

Серцева недостатність Heart failure

Гіпотонічна маткова кровотеча Hypotonic uterine bleeding

Бронхіальна астма Bronchial asthma

Анафілактичний шок Anaphylactic shock

146 / 200
Необхідно провести електричну дефібриляцію дитині з фібриляцією шлуночків. Маса тіла становить близько 10 кг. Оберіть вірну потужність розряду для першої дефібриляції: It is necessary to perform electrical defibrillation on a child with ventricular fibrillation. The body weight is about 10 kg. Choose the correct discharge power for the first defibrillation:

4 Дж 4 J

40 Дж 40 J

5 Дж/кг 5 J/kg

20 Дж 20 J

80 Дж 80 J

147 / 200
Хворий 83-х років, що перебуває в палаті інтенсивної терапії з приводу загострення ішемічної хвороби серця, раптом втратив свідомість. Зафіксовано клінічну смерть. Вкажіть цільові значення сатурації киснем артеріальної крові (Sp02) постраждалого після відновлення спонтанного кровообігу: An 83-year-old patient in the intensive care unit due to an exacerbation of coronary heart disease suddenly lost consciousness. Clinical death was recorded. Specify the target values ​​of arterial oxygen saturation blood (Sp02) of the victim after restoration of spontaneous blood circulation:

96-98% 96-98%

86-90% 86-90%

100% 100%

91-93% 91-93%

85% 85%

148 / 200
У хворого похилого віку діагностовано клінічну смерть. Почато реанімаційні заходи. Під час виконання непрямого масажу серця лікар відчув характерних хруст, резистентність грудної клітки істотно зменшилась. Яка подальша тактика лікаря? Clinical death was diagnosed in an elderly patient. Resuscitation measures were initiated. During indirect heart massage, the doctor felt characteristic crunches, the resistance of the chest significantly decreased. What are the doctor's further tactics ?

Припинення реанімаційних заходів Stop resuscitation

Продовження тільки штучного дихання Continuation of artificial respiration only

Поворот хворого на бік Turn the patient on his side

Продовження реанімації, непрямого масажу серця Continuation of resuscitation, indirect heart massage

Перекладання хворого на м’яку поверхню Transferring the patient to a soft surface

149 / 200
У хворої 47-ми років 3 дні тому з’явились помірний біль у горлі, кашель, захриплість голосу, температура - 37,8°С, поступово наросла інспіраторна задишка, кашель став афонічним. Об’єктивно: стан важкий, неспокійна, шкіра бліда з ціанотичним відтінком. В акті дихання бере участь допоміжна мускулатура. На мигдаликах, з розповсюдженням на піднебінні дужки, нальоти сірувато-білого кольору. Яку терапію слід призначити? 3 days ago, a 47-year-old patient developed a moderate pain in the throat, cough, hoarseness, temperature - 37.8°C, gradually increased inspiratory shortness of breath, cough has become aphonic. Objectively: the condition is severe, restless, the skin is pale with a cyanotic shade. Auxiliary muscles are involved in the act of breathing. Grayish-white plaques on the tonsils, spreading to the palatal arches. What therapy should be prescribed?

Глюкокортикостероїди, антибіотики Glucocorticosteroids, antibiotics

ПІВЛ, специфічна сироватка, антибіотики PIVL, specific serum, antibiotics

ГБО, специфічна сироватка HBO, specific serum

Селективні β2- адреноміметики, специфічна сироватка Selective β2-adrenomimetics, specific serum

Специфічна сироватка, антибіотики Specific serum, antibiotics

150 / 200
У хворої 56-ти років, яка знаходиться на лікуванні в терапевтичному відділенні з приводу хронічного бронхіту, раптово з’явився різкий біль у лівій половині грудної клітки, задишка, біль у ділянці серця, серцебиття, сухий надсадний кашель. Об’єктивно: виражений акроціаноз, розширення міжреберних проміжків та різке обмеження дихальної екскурсії. Перкуторно - зліва тимпаніт. Аускультативно - різке ослаблення дихання. Яку невідкладну допомогу слід надати хворій? A 56-year-old patient, who is being treated in the therapeutic department for chronic bronchitis, suddenly developed a sharp pain in the left half of the chest, shortness of breath, pain in the area of ​​the heart, palpitations, dry cough. Objectively: marked acrocyanosis, widening of the intercostal spaces and a sharp limitation of the respiratory excursion. Percussion - left tympanitis. Auscultation - a sharp weakening of breathing. What emergency help should be given to the patient?

Забезпечити доступ кисню Ensure access to oxygen

Введення анальгетиків Introduction of analgesics

Терміново транспортувати до відділєння торакальної хірургії Urgent transport to the department of thoracic surgery

Пункція плевральної порожнини Puncture of the pleural cavity

Введення еуфіліну Introducing Euphilin

151 / 200
У хворого періодично виникають напади клонічних судом у лівій руці, які іноді розповсюджуються на ліву половину обличчя. Напад триває 5 хвилин, після чого відмічається слабкість у лівій руці, яка поступово зникає впродовж 2-3 годин. Який варіант судомного синдрому наведено? The patient periodically has attacks of clonic convulsions in the left hand, which sometimes spread to the left half of the face. The attack lasts 5 minutes, after which there is weakness in the left hand, which gradually disappears within 2-3 hours. What variant of the convulsive syndrome is given?

Напад Кожевніковської епілепсії Attack of Kozhevnikov's epilepsy

Напад Джексоновської епілепсії Attack of Jacksonian epilepsy

Генералізований епілептичний напад Generalized epileptic seizure

Малий епілептичний напад Minor seizure

Напад скроневої епілепсії Attack of temporal lobe epilepsy

152 / 200
На МПП доставлений сержант. Знаходився у групі солдат з ознаками променевої хвороби. Скарги на головний біль, нудоту. Боєць збуджений, тривожний, вимагає повного обстеження для підтвердження ураження іонізуючим опроміненням, стан задовільний, у присутності лікаря з’являються гикавка та позиви на блювання, посилюється збудження, вираз страждання на обличчі, хитка хода. Який попередній діагноз? A sergeant was brought to the MPP. He was in a group of soldiers with signs of radiation sickness. Complaints of headache, nausea. The fighter is excited, anxious, requires a full examination to confirm ionizing radiation damage radiation, the condition is satisfactory, in the presence of the doctor, hiccups and urges to vomit appear, excitement increases, expression of suffering on the face, shaky gait. What is the previous diagnosis?

Істерична реакція Hysterical reaction

Гостра променева хвороба І ступеня Acute radiation sickness of the 1st degree

Алкогольне сп’яніння Alcohol intoxication

Струс головного мозку Concussion

Інтоксикація наркотичними речовинами Drug intoxication

153 / 200
Під час проведення бойових дій містечко Н. піддалося масивному обстрілу установками ”Град” При масовому поступленні з території обстрілу поранених і хворих до медичної роти бригади, у сортувально-евакуаційному відділенні було проведено евакуаційнотранспортне медичне сортування. Які ще види медичного сортування повинні бути проведені в медичній роті бригади? During the hostilities, the town of N. was subjected to massive shelling by 'Grad' installations. During the mass influx from the shelling area of ​​the wounded and sick to the medical company of the brigade, in the sorting and evacuation center evacuation transport medical sorting was carried out in the department. What other types of medical sorting should be carried out in the medical company of the brigade?

Спеціалізоване Specialized

Діагностичне, прогностичне Diagnostic, prognostic

Кваліфіковане Qualified

Внутрішньопунктове Intrapoint

Вибіркове Selective

154 / 200
Після опромінення до госпіталю доставлений молодий чоловік 26-ти років. Показник індивідуального дозиметра 5 Гр. Скарги на різку загальну слабкість, біль голови, нудоту, повторне блювання. Об’єктивно: на шкірі обличчя еритема, Ps-100/хв., АТ- 90/60 мм рт.ст., блювання продовжується. В якому клінічному періоді гострої променевої хвороби знаходиться хворий? After radiation, a 26-year-old young man was taken to the hospital. The reading of the individual dosimeter was 5 Gy. Complaints of sudden general weakness, headache, nausea, repeated vomiting. About 'objectively: erythema on the skin of the face, Ps-100/min., BP- 90/60 mm Hg, vomiting continues. In what clinical period of acute radiation sickness is the patient?

Період первинної реакції Primary response period

Період вторинної реакції Secondary response period

Період запалення Inflammation period

Період розпалу Heating period

Період виражених клінічних проявів Period of pronounced clinical manifestations

155 / 200
У хворого 27-ми років через 1 годину після ураження отруйною речовиною розвинулася наступна симптоматика: тривалий кашель з виділенням слизу, утруднене дихання внаслідок бронхоспазму. Яка отруйна речовина може викликати дану симптоматику? A 27-year-old patient developed the following symptoms 1 hour after exposure to a poisonous substance: prolonged cough with mucus secretion, difficulty breathing due to bronchospasm. What poisonous substance can cause this symptom?

Адамсит Adamsite

Оксид вуглецю Carbon monoxide

Фосген Phosgene

Дибензосазепін Dibenzozazepine

Зарин Sarin

156 / 200
Хворого 37-ми років було поранено колючим предметом в плече. Об’єктивно: на медіальній поверхні у верхній третині правого плеча є глибока різана рана з пульсуючою струйною кровотечею. Пульсація на a. radialis відсутня. Визначте обсяг надання першої медичної допомоги: A 37-year-old patient was injured by a sharp object in the shoulder. Objectively: on the medial surface in the upper third of the right shoulder there is a deep cut wound with pulsating jet bleeding. There is no pulsation on the a. radialis. Determine the extent of providing first aid:

Накладання джгута дистальніше рівня кровотечі Applying a tourniquet distal to the bleeding level

Накладання стискаючої пов’язки Applying compression bandage

Накладання джгута проксимальніше рівня кровотечі Applying a tourniquet proximal to the bleeding level

Введення гемостатичних препаратів Introduction of hemostatic drugs

Холод на місце кровотечі Cold to the place of bleeding

157 / 200
До терапевтичного відділення надійшов чоловік 30-ти років зі скаргами на зниження гостроти зору, неприємні відчуття в ділянці перенісся. Одночасно з’явилися нежить з рясними рідкими виділеннями, слинотеча, стиснення у грудній клітці, утруднення дихання, кашель. Об’єктивно: різке звуження зіниць, відсутність зіничних реакцій на світло. Ps- 100/хв. Діагностовано отруєння ФОС. Який лікувальний засіб першочерговий в цьому випадку? A 30-year-old man came to the therapeutic department with complaints of reduced visual acuity, unpleasant sensations in the area of ​​the bridge of the nose. At the same time, a runny nose with abundant liquid secretions, drooling appeared , tightness in the chest, difficulty breathing, cough. Objectively: sharp narrowing of the pupils, lack of pupillary reactions to light. Ps- 100/min. FOS poisoning was diagnosed. What is the first-line treatment in this case?

Преднізолон Prednisone

Супрастин Suprastin

Розчин левоміцетину Solution of chloramphenicol

Атропін Atropine

Адреналін Adrenaline

158 / 200
Під час обстрілу в зоні бойових дій, у солдата з’явились наступні симптоми: став тривожним, надмірно метушливим, гіперактивним, не виконував накази командира, неадекватно реагував на навколишніх. Впродовж двох годин стан покращився, симптоми стали менш вираженими, пам’ять на події, що відбувались, частково втрачена. Який найбільш імовірний діагноз? During the shelling in the combat zone, the soldier developed the following symptoms: he became anxious, excessively fussy, hyperactive, did not follow the commander's orders, reacted inappropriately to others . Within two hours, the condition improved, the symptoms became less pronounced, the memory of the events that took place was partially lost. What is the most likely diagnosis?

Посттравматичний стресовий розлад Post-traumatic stress disorder

Гостра реакція на стрес Acute reaction to stress

Гострий поліморфний психотичний розлад Acute polymorphic psychotic disorder

Розлад адаптації Adaptation disorder

Контузія Contusion

159 / 200
У постраждалого з переламом стегнової кістки на 3-ю добу після травми раптово погіршився стан: втрата свідомості, збудження, задишка, ціаноз шкіри. Який найбільш імовірний діагноз? On the 3rd day after the injury, the victim with a fractured femur suddenly became worse: loss of consciousness, agitation, shortness of breath, cyanosis of the skin. What is the most likely diagnosis?

Септичний шок Septic shock

Жирова емболія легеневої артерії Fat embolism of the pulmonary artery

Гостра крововтрата Acute blood loss

Гангрена кінцівки Gangrene of limb

Напружений пневмоторакс Tension pneumothorax

160 / 200
Пацієнт 42-х років, оглянутий лікарем-психіатром, перебуває в стані оглушення, що періодично змінюється тривогою, відчаєм, афективними спалахами, гіперактивністю. На запитання не реагує. Відомо, що стан виник через декілька хвилин після бойових дій та триває 2 години. Який імовірний діагноз? A 42-year-old patient examined by a psychiatrist is in a state of stupor, which periodically alternates with anxiety, despair, affective outbursts, hyperactivity. He does not respond to questions. It is known that the condition arose a few minutes after the hostilities and lasted for 2 hours. What is the probable diagnosis?

Параноїдна реакція Paranoid reaction

Тривожна реакція Anxious reaction

Конверсійна реакція Conversion response

Іпохондричний невроз Hypochondriac neurosis

Гостра реакція на стрес Acute reaction to stress

161 / 200
Лейтенант під час приземлення з парашутом відчув гострий біль в правому гомілковоступневому суглобі. Через 2 години дійшов до медичного пункту батальйону. Об’єктивно: виражений набряк тканин правого гомілково-ступневого суглоба, особливо в ділянці зовнішньої кісточки. Активні рухи в правому гомілково-ступневому суглобі обмежені, пасивні - різко болючі. Під час пальпації гострий біль в ділянці зовнішньої кісточки. Який об’єм допомоги? During landing with a parachute, the lieutenant felt a sharp pain in the right ankle joint. After 2 hours, he reached the battalion medical center. Objectively: severe swelling of the tissues of the right ankle foot joint, especially in the area of ​​the outer bone. Active movements in the right ankle-foot joint are limited, passive - sharply painful. During palpation, sharp pain in the area of ​​the outer bone. What is the amount of help?

Знеболювання, транспортування в лікувальний заклад Anesthesia, transportation to a medical facility

Знеболювання, транспортна іммобілізація, транспортування на етап кваліфікованої допомоги Anesthesia, transport immobilization, transportation to the stage of skilled care

Іммобілізація, транспортування в лікувальний заклад Immobilization, transportation to a medical facility

Іммобілізація, транспортування на етап спеціалізованої медичної допомоги Immobilization, transportation to the stage of specialized medical care

Транспортна іммобілізація, транспортування в лікувальний заклад Transport immobilization, transportation to a medical facility

162 / 200
Хворий 18-ти років, скаржиться на слабкість, мерехтіння в очах, запаморочення, випорожнення темного кольору. Виразкового анамнезу немає. Об’єктивно: шкіра та слизові оболонки бліді, Ps- 112/хв., АТ-90/60 мм рт.ст. У крові: НЬ- 86 г/л. Яка тактика лікаря? An 18-year-old patient complains of weakness, flickering in the eyes, dizziness, dark-colored stools. There is no history of ulcers. Objectively: the skin and mucous membranes are pale , Ps- 112/min., BP-90/60 mm Hg. In blood: НБ- 86 g/l. What is the doctor's tactics?

Консервативна терапія в амбулаторних умовах Conservative therapy in outpatient settings

Призначення дієти Мейленграхта Meilengracht diet appointment

Спостереження за хворим в умовах поліклініки Observation of the patient in the clinic

Термінова госпіталізація в хірургічний стаціонар Urgent hospitalization in a surgical hospital

Госпіталізація в гастроентерологічне відділення Hospitalization in the gastroenterology department

163 / 200
Пацієнт 74-х років протягом останніх 5-ти років страждає на доброякісну гіперплазію простати. 4 доби тому, після вживання алкоголю, виникла гостра затримка сечовипускання. На догоспітальному етапі двічі на добу катетеризували сечовий міхур металевим катетером. Під час огляду: придаток правого яєчка збільшений, ущільнений, болючий, є гнійні виділення з уретри. Який вид невідкладної допомоги слід обрати? A 74-year-old patient has been suffering from benign prostatic hyperplasia for the past 5 years. 4 days ago, after drinking alcohol, an acute urinary retention occurred. At the pre-hospital stage twice a day, the bladder was catheterized with a metal catheter. During the examination: the right epididymis is enlarged, compacted, painful, there is purulent discharge from the urethra. What type of emergency care should be chosen?

Встановлення постійного уретрального катетера Indwelling urethral catheter

Мікрохвильова термотерапія простати Microwave thermotherapy of the prostate

Трансуретральна резекція Transurethral resection

Встановлення інтрапростатичного стента Installation of an intraprostatic stent

Епіцистостомія в ургентному порядку Urgent epicystostomy

164 / 200
Хвора 52-х років в кардіологічному відділенні на фоні підвищення добової дози ди-гоксину стала тривожною, збудженою, казала, що знаходиться вдома, 'бачить' навколо себе багато мишей, намагалася їх повиганяти і вимагала від дочки, яка, начебто, була присутня, щоб та їй допомагала. Правильно називала своє ім’я та вік. В анамнезі психічних розладів не спостерігалося. Який найбільш імовірний діагноз? A 52-year-old patient in the cardiology department, against the background of an increase in the daily dose of digoxin, became anxious, excited, said that she was at home, 'sees' a lot around her mice, tried to chase them away and asked her daughter, who seemed to be present, to help her. She gave her name and age correctly. There was no history of mental disorders. What is the most likely diagnosis?

Інтоксикаційний делірій Intoxication delirium

Продуктивні сутінки Productive Twilight

Сенильний делірій Senile delirium

Пізній дебют шизофренії Late onset schizophrenia

Хвороба Альцгеймера Alzheimer's disease

165 / 200
Хворий 35-ти років, з чисельними аб-сцедуючими фурункулами. Через 2 години після введення цефтріаксону у хворого знизився артеріальний тиск до 70/40 мм рт.ст., тахікардія до 100/хв., ЧД- 22/хв. Яке ускладнення виникло у хворого? A 35-year-old patient with numerous abscessing boils. 2 hours after administration of ceftriaxone, the patient's blood pressure decreased to 70/40 mm Hg. , tachycardia up to 100/min., HR - 22/min. What complication did the patient experience?

Тромбоемболія легеневої артерії Thromboembolism of the pulmonary artery

Респіраторний дистрес-синдром Respiratory distress syndrome

Кровотеча з сальникової сумки Bleeding from the omentum bag

Артеріальна гіпотонія Arterial hypotension

Інфекційно-токсичний шок Infectious-toxic shock

166 / 200
У хворої 52-х років з наявним трансфу-зіологічним анамнезом одразу після початку внутрішньовенного введення свіжоза-мороженої плазми виникли непродуктивний кашель, бронхоспазм, задуха, нудота, болі у животі, гіпотонія, після чого наступила втрата свідомості. Які найбільш правильні лікувальні заходи? A 52-year-old patient with a history of transfusion history developed a non-productive cough, bronchospasm, suffocation, nausea, pain in the abdomen, hypotension, followed by loss of consciousness. What are the most appropriate medical measures?

В/в введення еуфіліну, гідрокортизону, інфузійна терапія, інгаляція кисню IV administration of euphilin, hydrocortisone, infusion therapy, oxygen inhalation

Негайна зупинка інфузії, в/в введення адреналіну, преднізолону, еуфіліну, інгаляція кисню Immediate stop of infusion, intravenous administration of epinephrine, prednisone, euphylline, oxygen inhalation

Джгут вище місця введення, обколювання розчином адреналіну, гідрокортизон в/в Tourniquet above the injection site, circumcision with adrenaline solution, IV hydrocortisone

Негайна зупинка інфузії, інтубація трахеї, ШВЛ Immediate stop of infusion, tracheal intubation, ventilator

Продовження інфузії, введення адрено-стимуляторів, глюкокортикостероїдів Continuation of infusion, introduction of adreno-stimulators, glucocorticosteroids

167 / 200
Хвора 19-ти років збуджена, поведінка неадекватна, свідомість сплутана. В анамнезі цукровий діабет типу 1 впродовж 3-х років. Отримує інсулінотерапію 60 ОД/добу. Об’єктивно: шкіра волога, холодна на дотик, гіперрефлексія, зіниці розширені, менінгеальних симптомів немає. Ps- 88/хв., АТ- 140/90 мм рт.ст. Глікемія - 2,3 ммоль/л, аглюкозурія. Який найбільш імовірний діагноз? A 19-year-old patient is excited, her behavior is inadequate, her consciousness is confused. She has a history of type 1 diabetes for 3 years. She receives insulin therapy 60 units/day. About objectively: the skin is moist, cold to the touch, hyperreflexia, the pupils are dilated, there are no meningeal symptoms. Ps- 88/min., BP- 140/90 mm Hg. Glycemia - 2.3 mmol/l, aglucosuria. Which is the most probable diagnosis?

Гіпоглікемічна кома Hypoglycemic coma

Молочнокисла кома Lactic coma

Кетоацидотична кома Ketoacidotic coma

Гостре порушення мозкового кровообігу Acute cerebrovascular accident

Гіперосмолярна кома Hyperosmolar coma

168 / 200
Бригадою ЕМД оглянуто постражда-лого в результаті ДТП 40-ка років: травматичне пошкодження верхньої щелепи, пульс на сонній артерії відсутній, екскурсій грудної клітки немає. Прийнято рішення про проведення СЛР Який спосіб ШВЛ доцільно використати в даному випадку? The EMD team examined a 40-year-old victim of a road accident: traumatic damage to the upper jaw, no pulse on the carotid artery, no chest excursions. A decision was made to conducting CPR What type of ventilator should be used in this case?

ШВЛ ”рот до рота” Mouth to mouth ventilator

ШВЛ за Сільвестром Sylvester ventilator

ШВЛ ”рот до носа” Mouth to nose ventilator

ШВЛ за Холдером-Нільсеном Holder-Nielsen ventilation

ШВЛ ”одночасно і до рота, і до носа” Ventilation 'to the mouth and nose at the same time'

169 / 200
Чоловік 44-х років під час роботи на городі в спекотний день відчув головний біль, запаморочення, нудоту, шум у вухах, порушився зір. Об’єктивно: шкіра обличчя гіперемована, температура - 39oC, дихання часте, Ps- 110/хв., АТ- 110/70 мм рт.ст. Який діагноз найбільш імовірний? A 44-year-old man, while working in the garden on a hot day, experienced a headache, dizziness, nausea, tinnitus, and impaired vision. Objectively: skin the face is hyperemic, temperature - 39oC, breathing is frequent, Ps - 110/min., BP - 110/70 mm Hg. What is the most probable diagnosis?

Гостре респіраторне захворювання Acute respiratory disease

Колапс Collapse

Вегетативний криз Vegetative crisis

Тепловий удар середньої важкості Moderate heat stroke

Мігрень Migraine

170 / 200
Чоловік 57-ми років після навантаження відчув різко виражену задуху, страх смерті. Об’єктивно: дихання клокочуче на відстані, кашель з рожевою піною. Шкіра холодна, волога, акроціаноз, над легенями дрібнопухирчасті хрипи. АТ- 100/70 мм рт.ст., Ps100 /хв., ниткоподібний. Яке ускладнення розвинулось? A 57-year-old man felt severe suffocation after exertion, fear of death. Objectively: breathing is gurgling in the distance, cough with pink foam. The skin is cold, moist , acrocyanosis, fine-bubble rales over the lungs. Blood pressure - 100/70 mmHg, Ps100/min., filamentous. What complication developed?

Крупозна пневмонія Croup pneumonia

Набряк легень Pulmonary edema

Гостра ниркова недостатність Acute renal failure

Інфаркт мозку Cerebral infarction

Напад бронхіальної астми Bronchial asthma attack

171 / 200
Чоловік 69-ти років скаржиться на головний біль та різке погіршення зору на ліве око. Лікарем швидкої медичної допомоги при огляді визначено: у свідомості, АТ-170/100 мм рт.ст., ЧСС- 100/хв. Менінгеальні симптоми, парези, розлади чутливості та інші чіткі неврологічні розлади відсутні. Визначте подальшу тактику ведення хворого: A 69-year-old man complains of a headache and sharp deterioration of vision in the left eye. The emergency medical doctor during the examination determined: conscious, AT-170/100 mm Hg, heart rate - 100/min. There are no meningeal symptoms, paresis, sensitivity disorders, and other clear neurological disorders. Determine the further tactics of patient management:

Призначення консультації психіатра Psychiatrist consultation appointment

Призначення консультації невролога Appointment of neurologist consultation

Призначення консультації окуліста Oculist consultation appointment

Госпіталізація у неврологічне відділення Hospitalization in the neurological department

Госпіталізація в офтальмологічне відділення Hospitalization in the ophthalmology department

172 / 200
Пацієнту 65-ти років, який хворіє на облітеруючий атеросклероз судин нижніх кінцівок, виконали операцію - ампутація ноги на рівні н/3 стегна. В зв’язку з вираженим больовим синдромом в післяопераційному періоді хворому тричі вводився омнопон. Які ускладнення можуть виникнуть у хворого в зв’язку з використанням великих доз наркотичних анальгетиків? A 65-year-old patient suffering from obliterating atherosclerosis of the vessels of the lower extremities underwent an operation - amputation of the leg at the level of n/3 of the thigh. In connection with the pronounced pain syndrome in the postoperative period, the patient was administered omnopon three times. What complications may occur in the patient in connection with the use of large doses of narcotic analgesics?

Гостра печінкова недостатність Acute liver failure

Зупинка серця Cardiac arrest

Зупинка дихання Stop breathing

Фібриляція шлуночків серця Ventricular fibrillation

Гостра ниркова недостатність Acute renal failure

173 / 200
Чоловіку на ліву ногу впала балка. При обстеженні виявлені набряк, гематома, деформація і патологічна рухливість в середній третині лівого стегна. Яка шина забезпечить оптимальну транспортну іммобілізацію? A beam fell on the man's left leg. The examination revealed swelling, hematoma, deformation and pathological mobility in the middle third of the left thigh. Which tire will provide optimal transport immobilization?

Шина Еланського Shyna Elansky

Шина Дітеріхса Tire of Dieterichs

Шина Чижова Shina Chizhova

Шина Крамера Kramer Tire

Шина Дельбе Syna Delbe

174 / 200
Жінка 25-ти років скаржиться на плаксивість, різкі зміни настрою, прискорене серцебиття, що зв’язує з втомою - місяць тому народила близнюків. З ранку відчула ”зупинки” серцевої діяльності. При огляді з боку внутрішніх органів патології не встановлено. АТ- 130/70 мм рт.ст., ЧСС- 115/хв., часті екстрасистоли, ЧДР- 17/хв. Щитоподібна залоза збільшена до другого ступеня, безболісна. Який найбільш імовірний попередній діагноз? A 25-year-old woman complains of tearfulness, sudden changes in mood, rapid heartbeat, which is associated with fatigue - she gave birth to twins a month ago. She felt 'stops' in the morning 'cardiac activity. No pathology was found during the examination of the internal organs. Blood pressure - 130/70 mm Hg, heart rate - 115/min, frequent extrasystoles, HRD - 17/min. The thyroid gland was enlarged to the second degree, painless. What is the most likely preliminary diagnosis?

Гострий вірусний тиреоїдит, гіпотиреоз Acute viral thyroiditis, hypothyroidism

Післяпологовий тиреоїдит, гіпотиреоз Postpartum thyroiditis, hypothyroidism

Підгострий вірусний тиреоїдит, гіпертиреоз Subacute viral thyroiditis, hyperthyroidism

Гострий вірусний тиреоїдит, гіпертиреоз Acute viral thyroiditis, hyperthyroidism

Післяпологовий тиреоїдит, гіпертиреоз Postpartum thyroiditis, hyperthyroidism

175 / 200
У дитини 3-х років, яка знаходиться у стаціонарі, на фоні гострої респіраторної інфекції 3 рази було блювання, непокоїть гострий біль у животі. Температура тіла -38,5oC. Шкіра бліда, суха. Дихання глибоке, ЧСС- 130/хв. Цукор крові - 4,0 ммоль/л. Який з діагнозів найбільш імовірний? A 3-year-old child, who is in a hospital, has vomited 3 times against the background of an acute respiratory infection, is worried about acute abdominal pain. The body temperature is -38, 5oC. The skin is pale, dry. Breathing is deep, heart rate - 130/min. Blood sugar - 4.0 mmol/l. Which of the diagnoses is the most probable?

Цукровий діабет Diabetes

Гострий апендицит Acute appendicitis

Діабетична кома Diabetic coma

Гострий гастроентерит Acute gastroenteritis

Ацетонемічний стан Acetonemic state

176 / 200
Відразу після народження немовля нерухоме, синє і не дихає. Відповіді на тактильну стимуляцію немає, ЧСС становить 40/хв. Який із перерахованих заходів найімовірніше відновить нормальний спонтанний серцевий ритм? Immediately after birth, the infant is motionless, blue, and not breathing. There is no response to tactile stimulation, heart rate is 40/min. Which of the following measures is most likely to restore a normal spontaneous heart rhythm ?

Введення адреналіну Injection of adrenaline

Штучна вентиляція легень Artificial lung ventilation

Додаткова тактильна стимуляція Additional tactile stimulation

Введення атропіну Atropine administration

Непрямий масаж серця Indirect heart massage

177 / 200
В приймальне відділення звернулися батьки з хлопчиком 8-ми років після укусу бджоли. Об’єктивно: виражений набряк у ділянці верхньої щелепи з обох боків, верхньої губи, гіперемія і свербіж. Який найбільш імовірний діагноз? Parents with an 8-year-old boy came to the emergency department after a bee sting. Objectively: severe swelling in the area of ​​the upper jaw on both sides, upper lip, hyperemia and itching. What is the most likely diagnosis?

Атопічний дерматит Atopic dermatitis

Анафілактичний шок Anaphylactic shock

Кропив’янка Hives

Стрептодермія Streptoderma

Набряк Квінке Quincke edema

178 / 200
Ви надаєте допомогу постраждалій дитині з електротравмою. Постраждалий непритомний, тримається за електричний провід. З чого необхідно починати надавати допомогу? You are providing assistance to an injured child with an electric shock. The victim is unconscious, holding on to an electric wire. Where should you start providing assistance?

Провести непрямий масаж серця Perform an indirect heart massage

Провести ШВЛ Perform ventilation

Звільнити постраждалого від дії електричного струму Release the electrocuted victim

Дати понюхати нашатирний спирт Let ammonia smell

Викликати швидку допомогу Call an ambulance

179 / 200
У літньої людини 72-х років під час коронарографії та стентування вінцевих артерій з приводу ішемічної хвороби серця на кардіомоніторі відмічена фібриляція шлуночків. Що первинно необхідно провести хворому? In a 72-year-old elderly person, during coronary angiography and coronary artery stenting due to ischemic heart disease, ventricular fibrillation was noted on the cardiomonitor. What should the patient do first?

Дефібриляція Defibrillation

Катетеризація сечового міхура Catheterization of urinary bladder

Інтубація трахеї Tracheal intubation

Зондування шлунка Stomach sounding

Кава-катетеризація Kava-catheterization

180 / 200
Перша медична допомога (само- та взаємодопомога) при радіаційних ураженнях передбачає усунення або послаблення початкових ознак променевої хвороби. З цією метою особовий склад Збройних Сил безпосередньо після вибуху для профілактики первинної реакції бере з індивідуальної аптечки (одну таблетку): First medical aid (self- and mutual aid) for radiation injuries involves eliminating or weakening the initial signs of radiation sickness. For this purpose, the personnel of the Armed Forces immediately after the explosion for prevention primary reaction is taken from an individual first-aid kit (one tablet):

Доксицикліну гідрохлорид Doxycycline hydrochloride

Вібрацин Vibracin

Етаперазин Etaperazin

Будаксим Budaksim

Цистамін Cystamine

181 / 200
В ході бойових дій боєць отримав вогнепальне поранення черевної стінки. На полі бою була надана домедична допомога. При надходженні пораненого до мобільного госпіталю виявлені ознаки внутрішньочеревної кровотечі. До якої сортувально-евакуаційної групи необхідно віднести цього постраждалого? During the hostilities, a fighter received a gunshot wound to the abdominal wall. On the battlefield, first aid was provided. When the wounded man arrived at the mobile hospital, signs of intra-abdominal bleeding were detected. - the evacuation group should this victim be assigned to?

Першої First

Другої Second

Третьої Third

П’ятої On the fifth

Четвертої Fourth

182 / 200
Військовий під час мінометного обстрілу був засипаний землею до грудей на 6 годин. По латеральній поверхні правого стегна та в районі кульшового суглоба щільна припухлість, зниження чутливості шкіри. Діагностовано синдром тривалого стиснення. Яку інфузійну рідину доцільно призначити для профілактики ниркової недостатності? During the mortar shelling, the soldier was covered with earth up to his chest for 6 hours. On the lateral surface of the right thigh and in the area of ​​the hip joint, dense swelling, decreased skin sensitivity. The syndrome was diagnosed long-term compression. Which infusion fluid should be prescribed for the prevention of renal failure?

10% розчин кальцію хлориду 10% calcium chloride solution

10% розчин натрію хлориду 10% sodium chloride solution

0,45% розчин натрію хлориду 0.45% sodium chloride solution

5% розчин глюкози 5% glucose solution

4% розчин натрію бікарбонату 4% sodium bicarbonate solution

183 / 200
У військовослужбовця в зоні бойових дій має місце вивих великого пальця лівої руки. До якої сортувальної категорії з її візуальним позначенням буде віднесений потерпілий? A military serviceman in a combat zone has a sprain of his left thumb. To which sorting category with its visual designation will the victim be assigned?

І (червоний колір) And (red color)

IV (темно-фіолетовий колір) IV (dark purple color)

ІІІ (зелений колір) III (green color)

V (синій колір) V (blue color)

ІІ (жовтий колір) II (yellow color)

184 / 200
Під час робочої зміни на молочноконсервному комбінаті виникла аварія холодильної установки з потраплянням в атмосферу приміщення парів аміаку. 16 робітників скаржаться на захриплість голосу, загрудинний біль, кашель з кров’янистим харкотинням. Об’єктивно: ціаноз, субіктеричність склер. У крові: метгемоглобінемія. Які правильні дії при наданні допомоги постраждалим на різних етапах? During the work shift at the milk canning plant, a refrigeration plant accident occurred with ammonia vapors entering the room atmosphere. 16 workers complain of hoarseness of voice, chest pain, coughing up blood' purulent sputum. Objectively: cyanosis, subicteric sclera. In the blood: methemoglobinemia. What are the correct actions when providing assistance to victims at different stages?

Виведення постраждалих із зони ураження. Гаряче питво. Гіпербарична оксигенація Removal of victims from the affected area. Hot drink. Hyperbaric oxygenation

ШВЛ. Дипіроксим в/м VENTILATOR. Dipiroxim IV

Виведення постраждалих із зони ураження. Атропін п/ш. Промивання шлунка з активованим вугіллям Removal of victims from the affected area. Atropine p/sh. Gastric lavage with activated charcoal

Виведення постраждалих із зони ураження. Діцинон в/м Removal of victims from the affected area. Dicinon IV

Виведення постраждалих із зони ураження. Оксигенотерапія. Метиленовий синій в/в Removal of victims from the affected area. Oxygen therapy. Intravenous methylene blue

185 / 200
У хворого 47-ми років під час трансфузії еритроцитарної маси виникли занепокоєння, задишка, біль у попереку. Об’єктивно: почервоніння обличчя, ціаноз, Ps- 112/хв., АТ90 /60 мм рт.ст., сеча червонуватого кольору. Яка найбільш імовірна причина погіршення стану? A 47-year-old patient developed anxiety, shortness of breath, and back pain during transfusion of erythrocyte mass. Objectively: facial redness, cyanosis, Ps-112/ min., blood pressure 90 /60 mmHg, urine is reddish in color. What is the most likely cause of the deterioration?

Гостра дихальна недостатність Acute respiratory failure

Гостра серцева недостатність Acute heart failure

Переливання несумісної еритромаси Transfusion of incompatible erythromass

Надмірний обсяг інфузії Excessive volume of infusion

Гостра надниркова недостатність Acute adrenal insufficiency

186 / 200
Хворий 70-ти років отримав травму на вулиці. Відчув різкий біль в проксимальному відділі лівого плеча. Вкажіть оптимальний метод транспортної іммобілізації: A 70-year-old patient was injured on the street. He felt a sharp pain in the proximal part of the left shoulder. Specify the optimal method of transport immobilization:

Іммобілізація плеча драбинчатою шиною Immobilization of the shoulder with a ladder splint

Фіксація руки до тулуба Fixing the arm to the body

Вкласти хворого на ноші Put the patient on the stretcher

Іммобілізація плеча шиною Дітеріхса Shoulder immobilization with Dieterichs splint

Еластична пов’язка Elastic bandage

187 / 200
Чоловік 54-х років, хворий на ХІХС, був на стаціонарному лікуванні. Раптово знепритомнів, пульсація магістральних артеріальних судин не визначається, дихання відсутнє, мідріаз. Лікар почав закритий масаж серця та ШВЛ. На електрокардіограмі ізолінія. Який варіант зупинки кровообігу? A 54-year-old man suffering from chronic obstructive pulmonary disease was receiving inpatient treatment. He suddenly fainted, the pulsation of the main arterial vessels was not detected, there was no breathing, mydriasis. The doctor started closed heart massage and mechanical ventilation. An isoline on the electrocardiogram. What is the option of stopping blood circulation?

Пароксизмальна тахікардія Paroxysmal tachycardia

Асистолія Asystole

Фібриляція шлуночків Ventricular fibrillation

Тріпотіння передсердь Atrial flutter

Електромеханічна дисоціація Electromechanical dissociation

188 / 200
Під час дорожньо-транспортної пригоди постраждала дитина віком 10 років. Через 15 хвилин лікар швидкої допомоги поставив діагноз: відкритий перелом лівої стегнової кістки в нижній третині із зміщенням, артеріо-венозна кровотеча. Травматичний шок ІІ ступеня. Яка невідкладна первинна допомога? During a traffic accident, a 10-year-old child was injured. 15 minutes later, the emergency doctor made a diagnosis: open fracture of the left femur in the lower third with displacement, arterio -venous bleeding. Second-degree traumatic shock. What is the emergency first aid?

Новокаїнова блокада Novocaine blockade

Зупинка кровотечі Stop bleeding

Транспортна іммобілізація Transport immobilization

Знеболювання Pain relief

Накладання асептичної пов’язки Applying an aseptic bandage

189 / 200
У дівчинки 3-х років з тетрадою Фалло раптово виник напад, дитина стала неспокійною, підсилилась задишка. Об’єктивно: різкий ціаноз, ослаблений ІІ тон над легеневою артерією, грубий систолічний шум у третьому-четвертому міжребер’ї зліва, втрачає свідомість, з’являються судоми. Який стан виник у хворої? A 3-year-old girl with tetrad of Fallot suddenly had a seizure, the child became restless, shortness of breath increased. Objectively: sharp cyanosis, weakened II tone over the pulmonary artery , a rough systolic murmur in the third-fourth intercostal space on the left, loses consciousness, convulsions appear. What condition did the patient have?

Колапс Collapse

Задишково-ціанотичний криз Dyspnea-cyanotic crisis

Пневмоторакс Pneumothorax

Тромбоемболія легеневої артерії Thromboembolism of the pulmonary artery

Набряк легень Pulmonary edema

190 / 200
Хворий 62-х років раптово знепритомнів. Об’єктивно: шкірні покриви бліді, дихання відсутнє, пульс та артеріальний тиск не визначаються. На ЕКГ - фібриляція шлуночків. Тричі була проведена дефібриляція, ефекту немає. Введення якого препарату необхідно розпочати для підвищення ефекту дефібриляції? A 62-year-old patient suddenly fainted. Objectively: the skin is pale, there is no breathing, the pulse and blood pressure are not determined. On the ECG - ventricular fibrillation. Three times defibrillation was performed, there is no effect. Which drug must be administered to increase the effect of defibrillation?

Добутамін Dobutamine

Аміодарон Amiodarone

Урапідил Urapidil

Атропін Atropine

Дофамін Dopamine

191 / 200
До сімейного лікаря звернулась хвора 54-х років, зі скаргами на 'стартовий', ”блокадний” біль в колінних та кульшових суглобах, рухи обмежені, болючі. Хвора працює кухарем, підвищеного живлення. З анамнезу відомо, що мати хворої теж мала подібні скарги. Який метод діагностики є визначальним для встановлення діагнозу? A 54-year-old female patient came to the family doctor with complaints of 'starting', 'blocking' pain in the knee and hip joints, limited, painful movements. The patient works as a cook, with increased nutrition. It is known from the anamnesis that the patient's mother also had similar complaints. What diagnostic method is decisive for establishing the diagnosis?

Загальний аналіз крові General blood test

Загальний аналіз сечі General urinalysis

Біохімічне визначення рівня сечовини Biochemical determination of urea level

Рентгенографія X-ray

Артроскопія Arthroscopy

192 / 200
Пацієнту 70-ти років в зв’язку з недостатнім ефектом лікування артеріальної гіпертензії інгібітором АПФ у поєднанні із діуретиком, додатково призначений фізіотенс 0,4 мг/добу. Вночі, після швидкого підведення з ліжка, відчув запаморочення, нудоту, потемніння в очах, серцебиття, похолодання і оніміння кінцівок. АТ- 80/54 мм рт.ст. На ЕКГ ритм синусовий, ЧСС- 108/хв. Після введення мезатону стан пацієнта відносно швидко нормалізувався. Як розцінити цей епізод? A 70-year-old patient, due to the insufficient effect of the treatment of arterial hypertension with an ACE inhibitor in combination with a diuretic, was additionally prescribed physiotens 0.4 mg/day. At night , after quickly getting out of bed, felt dizziness, nausea, darkening of the eyes, palpitations, coldness and numbness of the limbs. Blood pressure - 80/54 mm Hg. On the ECG, the rhythm is sinus, heart rate - 108/min. After the introduction of mesaton, the patient's condition is relatively quickly recovered. How to evaluate this episode?

Гостра серцева недостатність Acute heart failure

Ортостатичний колапс Orthostatic collapse

Кардіогенний шок Cardiogenic shock

Гіповолемічний колапс Hypovolemic collapse

Пароксизм суправентрикулярної тахікардії Paroxysm of supraventricular tachycardia

193 / 200
У дитини 8-ми місяців раптово виникла обструкція верхніх дихальних шляхів, що спричинена стороннім тілом. Візуально стороннє тіло не видно. Назвіть порядок надання невідкладної допомоги в цьому випадку: An 8-month-old child suddenly developed upper airway obstruction caused by a foreign body. Visually, the foreign body is not visible. Name the order of emergency care in this case:

В позиції 'вершника' з головою, що опущена нище тулуба, по спині між лопатками виконати 4 удари проксимальною частиною долоні In the 'horseman' position with the head lowered below the body, perform 4 blows on the back between the shoulder blades with the proximal part of the palm

Натискання на живіт Abdominal pressure

Очистка пальцем верхніх дихальних шляхів Upper airway finger cleaning

Очистка верхніх дихальних шляхів за допомогою пінцета Меджилла Cleaning of the upper respiratory tract with Magill's tweezers

Очистка верхніх дихальних шляхів за допомогою зажима Келлі Cleaning the upper airway with a Kelly clamp

194 / 200
У відділення реанімації бригадою швидкої допомоги доставлена дитина 3-х років у непритомному стані. Під час проведення штучного дихання та непрямого масажу серця на ЕКГ реєструється шлуно-чкова брадикардія 50/хв. Якому із медикаментозних заходів необхідно надати перевагу? An unconscious 3-year-old child was brought to the intensive care unit by the ambulance team. During artificial respiration and indirect heart massage, ventricular bradycardia was registered on the ECG 50 /min. Which of the medicinal measures should be given priority?

Внутрішньовенне ведення гідрокарбонату натрію Intravenous administration of sodium bicarbonate

Електрична дефібриляція серця Electrical defibrillation of the heart

Внутрішньовенне введення адреналіну Intravenous administration of epinephrine

Внутрішньовенне ведення атропіну Intravenous administration of atropine

Електрична дефібриляція серця + введення адреналіну Electric heart defibrillation + adrenaline injection

195 / 200
Загальний стан доношеної новонародженої дитини середньої тяжкості, що зумовлено асфіксією середнього ступеня важкості. Маса при народженні 3500 г, довжина 54 см. Оцінка за шкалою Апгар на 1 хвилині 4 бали. Після проведення первинної реанімаційної допомоги з’явилося самостійне дихання, ЧСС- 110/хв., акроціаноз. Яка подальша дія лікаря у даній ситуації'? General condition of a full-term newborn child of moderate severity, due to asphyxia of moderate severity. Birth weight 3500 g, length 54 cm. Apgar score at 1 minute 4 points. After primary resuscitation, spontaneous breathing appeared, heart rate - 110/min., acrocyanosis. What is the further action of the doctor in this situation?

Відсмоктування слизу з верхніх дихальних шляхів Suction of mucus from the upper respiratory tract

Додаткова оксигенація Additional oxygenation

Допоміжна вентиляція легенів Assisted lung ventilation

Штучний масаж серця Artificial heart massage

Інтубація трахеї Tracheal intubation

196 / 200
Поранений з травматичною ампутацією правої верхньої кінцівки доставлений з поля бою в мобільний госпіталь у стані шоку. АТ- 50/0 мм рт.ст. На травмованій кінцівці - турнікет. Які першочергові заходи мають бути здійснені на цьому етапі? The wounded with a traumatic amputation of the right upper limb was brought from the battlefield to a mobile hospital in a state of shock. Blood pressure - 50/0 mm Hg. A tourniquet is on the injured limb . What priority measures should be taken at this stage?

Розпочати краплинну інфузію 0,9% розчину натрію хлориду у межах подвоєної добової потреби в рідині Start a drip infusion of 0.9% sodium chloride solution within twice the daily fluid requirement

Послабити турнікет. Якщо відновилась кровотеча, то ввести гемостатичні препарати і розпочати інфузію кристалоїдів Relax the tourniquet. If bleeding resumes, administer hemostatic drugs and start crystalloid infusion

Внутрішньовенно ввести 1 мл адреналіну. Повторювати кожні 5 хвилин для підтримки адекватного АТ Inject 1 ml of epinephrine intravenously. Repeat every 5 minutes to maintain adequate BP

Швидке відновлення ОЦК, адекватне знеболювання Quick recovery of BCC, adequate analgesia

Розпочати внутрішньовенну інфузію мезатону і норадреналіну Start intravenous infusion of mesatone and norepinephrine

197 / 200
Ураженого госпіталізовано до мобільного госпіталю із радіаційно зараженої зони в тяжкому стані зі скаргами на безперервне блювання, пронос, болі в животі. Об’єктивно: гіперемія шкіри обличчя, температура тіла - 38,7oC. ЧСС- 105/хв., АТ-90/65 мм рт.ст. Діагностовано гостру променеву хворобу. Яка форма гострої променевої хвороби найбільш імовірна? The affected person was hospitalized in a mobile hospital from a radiation-contaminated zone in a serious condition with complaints of continuous vomiting, diarrhea, abdominal pain. Objectively: facial hyperemia, temperature body - 38.7oC. Heart rate - 105/min., BP - 90/65 mm Hg. Acute radiation sickness was diagnosed. What form of acute radiation sickness is most likely?

Церебральна Cerebral

Кістково-мозкова Bone brain

Кишкова Intestinal

Перехідна форма Transitional form

Судинно-токсемічна Vascular-toxemic

198 / 200
В медичну роту бригади надійшов поранений в тяжкому стані. Скарги на болі у животі, нудоту, сухість в роті. Об’єктивно: Ps- 130/хв., АТ- 80/40 мм рт.ст., SaO2 -92%. Язик сухий, обкладений білим нальотом, живіт не бере участі у акті дихання, при пальпації напружений, болісний у всіх відділах, перистальтика поодинокими хвилями. Притуплення у пологих місцях черевної порожнини. Попередній діагноз: внутрішньочеревна кровотеча. В який підрозділ госпіталізувати постраждалого? A wounded man was admitted to the brigade's medical company in a serious condition. Complaints of abdominal pain, nausea, dry mouth. Objectively: Ps- 130/min., Blood pressure - 80/40 mm Hg, SaO2 -92%. The tongue is dry, covered with a white coating, the stomach does not participate in the act of breathing, on palpation it is tense, painful in all parts, peristalsis in single waves. Dullness in the soft parts of the abdominal cavity Preliminary diagnosis: intra-abdominal bleeding. In which unit should the victim be hospitalized?

Госпітальне відділення Hospital department

Ізолятор Isolator

Операційна Operational

Відділення спецобробки Department of special processing

ВРІТ VRIT

199 / 200
У військовослужбовця кульове поранення середньої третини правого стегна з гнійносептичними ускладненнями. При посіві крові виявлено золотистий стафілокок. Додались скарги на болі у серці, задуху, перебої у роботі серця. Стан важкий, шкіра бліда, акроціаноз, температура - 37,8oC, Ps-106/хв., AT- 90/70 мм рт.ст. Тони серця ослаблені, екстрасистоли, систолічний шум на верхівці. Набряки нижніх кінцівок. Дихання везикулярне. З чим пов’язано погіршення стану? A military serviceman has a bullet wound in the middle third of his right thigh with purulent-septic complications. Staphylococcus aureus was detected during a blood culture. Complaints of heart pain, suffocation, and heart failure were added. The condition is severe, the skin is pale, acrocyanosis, temperature - 37.8oC, Ps-106/min., AT- 90/70 mm Hg. Heart tones are weakened, extrasystoles, systolic murmur at the apex. Edema of the lower extremities. Vesicular breathing. With what is the reason for the worsening of the condition?

Перикардит Pericarditis

Ревматизм Rheumatism

Стафілококовий міокардит Staphylococcal myocarditis

Хронічна хвороба нирок Chronic kidney disease

Набряк легень Pulmonary edema

200 / 200
Пацієнт 25-ти років отримав відкритий перелом обох кісток правої гомілки при вибуху міни. На етапі кваліфікованої медичної допомоги проводиться первинна хірургічна обробка рани. Яка хірургічна маніпуляція категорично заборонена? A 25-year-old patient received an open fracture of both bones of the right tibia during a mine explosion. At the stage of qualified medical care, primary surgical treatment of the wound is performed. Which surgical manipulation is categorically prohibited?

Видалення зруйнованих тканин Removing destroyed fabrics

Видалення сторонніх тіл Remove foreign bodies

Глухий шов рани Deaf wound suture

Проточне дренування рани Flow wound drainage

Широке розсічення рани Wide dissection of the wound